Superdeterminism And Sabine Hossenfelder

57 views
Skip to first unread message

John Clark

unread,
Dec 20, 2021, 7:01:05 PM12/20/21
to 'Brent Meeker' via Everything List

Brent Meeker meeke...@gmail.com

 Yes, it's empirically supported; So's the Schroedinger equation.  But it's part of the application of the Schroedinger equation.  It's not in the equation itself. 

> I don't know what you mean by that. 
> It's the projection postulate in the Copenhagen interpretation that applies the Born rule.  In MWI it's the Born rule plus some kind of self-locating uncertainty to allow for the probabilistic observations.  So those are things not in the Schroedinger equation.

I don't know how you figure that. It has been mathematically proven that the Born rule is the only way to get probabilities out of Schrodinger's equation such that all the probabilities add up to 1. And Schrodinger says an electron wave can be in any location, and in a camera/electron wave a camera will observe the electron being in every location, and in a Brent Meeker/camera/electron wave there will be a  Brent Meeker for every camera that sees an electron in every location.

> No, you can't observe the Born rule to be true any more (or less) than you can observe Schroedinger's equation to be true.

Nonsense! Every quantum physicist alive believes the Born rule is valid and they use it every day, and the reason they're so confident is because the Born rule has always conform with observations and all empirical tests , so it doesn't need a seal of approval  from a theory for us to think it's true, but a theory may need a seal of approval from the Born Rule to convince us that a theory is true. That's because observation always outranks theory.  

John K Clark


Brent Meeker

unread,
Dec 20, 2021, 7:50:12 PM12/20/21
to everyth...@googlegroups.com


On 12/20/2021 4:00 PM, John Clark wrote:

Brent Meeker meeke...@gmail.com

 Yes, it's empirically supported; So's the Schroedinger equation.  But it's part of the application of the Schroedinger equation.  It's not in the equation itself. 

> I don't know what you mean by that. 
> It's the projection postulate in the Copenhagen interpretation that applies the Born rule.  In MWI it's the Born rule plus some kind of self-locating uncertainty to allow for the probabilistic observations.  So those are things not in the Schroedinger equation.

I don't know how you figure that. It has been mathematically proven that the Born rule is the only way to get probabilities out of Schrodinger's equation such that all the probabilities add up to 1.

I'm well aware of that, and that's why I phrased it as "to allow for the probabilistic observations".  MWI is completely deterministic, including the prediction that all possibilities occur.  So you have to have some assumption to get probabilities, such that one thing happens and others don't.  MWI finesses this by saying that you observe all possible outcomes...but in other worlds.  But the mechanism of this splitting, when and where it happens, is as just as hand wavy as Copenhagen's projection postulate.  It's of the form: This must be how it works because that will give the right answer.  That's not wrong...but neither is it an improvement.


And Schrodinger says an electron wave can be in any location, and in a camera/electron wave a camera will observe the electron being in every location, and in a Brent Meeker/camera/electron wave there will be a  Brent Meeker for every camera that sees an electron in every location.

That's like saying every horse in the gate is a possible winner of the Kentucy derby.  But that doesn't get you to probabilities without an assumption that one an only one will win.  Everett wants to avoid that assumption...which then takes self-locating uncertainty to make it consistent with probabilistic observations.


> No, you can't observe the Born rule to be true any more (or less) than you can observe Schroedinger's equation to be true.

Nonsense! Every quantum physicist alive believes the Born rule is valid and they use it every day, and the reason they're so confident is because the Born rule has always conform with observations and all empirical tests , so it doesn't need a seal of approval  from a theory for us to think it's true, but a theory may need a seal of approval from the Born Rule to convince us that a theory is true. That's because observation always outranks theory. 

But observation is always finite, while theories claim infinite applicability.  Newton's mechanics is also used everyday, with confidence.  I didn't say theory made it true.  Theory only shows the Born rule is consistent with Hilbert space. 

Brent

Jesse Mazer

unread,
Dec 20, 2021, 7:53:03 PM12/20/21
to everyth...@googlegroups.com
But one of the big selling points of the MWI is to give some sort of objective picture of reality in which "measurements" have no distinguished role, but are simply treated using the usual rules of quantum interactions. If you have to say "OK, I believe in the MWI plus Born rule for measurements" with there being no dynamical definition of what qualifies as a measurement, where the moments we call 'measurements' are just something we feed into the theory on a know-it-when-I-see-it basis, then this claim to objectivity is lost and it's not clear what theoretical appeal it has over the Copenhagen interpretation. 

Personally I still lean towards some version of the MWI being true mainly because you can come up with a toy model with MWI-style splitting that deals with Bell style experiments in a way that preserves locality but doesn't require hidden variables (see https://www.mdpi.com/1099-4300/21/1/87/htm ) but I see it as a sort of work in progress rather than a complete interpretation.


 
 

John K Clark


--
You received this message because you are subscribed to the Google Groups "Everything List" group.
To unsubscribe from this group and stop receiving emails from it, send an email to everything-li...@googlegroups.com.
To view this discussion on the web visit https://groups.google.com/d/msgid/everything-list/CAJPayv3QV08JYEScbA6V2kQq78KvmvvBLPAtqd8MNNMAUcH8VQ%40mail.gmail.com.

Bruce Kellett

unread,
Dec 20, 2021, 8:10:15 PM12/20/21
to Everything List
On Tue, Dec 21, 2021 at 11:53 AM Jesse Mazer <laser...@gmail.com> wrote:
On Mon, Dec 20, 2021 at 7:01 PM John Clark <johnk...@gmail.com> wrote:

Brent Meeker meeke...@gmail.com

 Yes, it's empirically supported; So's the Schroedinger equation.  But it's part of the application of the Schroedinger equation.  It's not in the equation itself. 

> I don't know what you mean by that. 
> It's the projection postulate in the Copenhagen interpretation that applies the Born rule.  In MWI it's the Born rule plus some kind of self-locating uncertainty to allow for the probabilistic observations.  So those are things not in the Schroedinger equation.

I don't know how you figure that. It has been mathematically proven that the Born rule is the only way to get probabilities out of Schrodinger's equation such that all the probabilities add up to 1. And Schrodinger says an electron wave can be in any location, and in a camera/electron wave a camera will observe the electron being in every location, and in a Brent Meeker/camera/electron wave there will be a  Brent Meeker for every camera that sees an electron in every location.

> No, you can't observe the Born rule to be true any more (or less) than you can observe Schroedinger's equation to be true.

Nonsense! Every quantum physicist alive believes the Born rule is valid and they use it every day, and the reason they're so confident is because the Born rule has always conform with observations and all empirical tests , so it doesn't need a seal of approval  from a theory for us to think it's true, but a theory may need a seal of approval from the Born Rule to convince us that a theory is true. That's because observation always outranks theory.

But one of the big selling points of the MWI is to give some sort of objective picture of reality in which "measurements" have no distinguished role, but are simply treated using the usual rules of quantum interactions.

At one time, that might have been a point on which to prefer MWI over Bohr's version of the CI, but that is no longer true. Modern collapse theories do not have to distinguish particular "measurement" events, and do not have to assume a classical superstructure . In modern fGRW, for example, everything can be treated as quantum, and the theory is completely objective.

fGRW has the added advantage that it is an inherently stochastic theory. Probability is treated as a primitive notion that is not based on anything else. MWI struggles with the concept of probability, and while it has to reject a frequentist basis for probability, it cannot really supply anything else. Self-locating uncertainty does not, in itself, serve to define probability. You have to have some notion of a random selection from a set, and that is not available in either the Schrodinger equation or in self-locating uncertainty.


If you have to say "OK, I believe in the MWI plus Born rule for measurements" with there being no dynamical definition of what qualifies as a measurement, where the moments we call 'measurements' are just something we feed into the theory on a know-it-when-I-see-it basis, then this claim to objectivity is lost and it's not clear what theoretical appeal it has over the Copenhagen interpretation. 

Personally I still lean towards some version of the MWI being true mainly because you can come up with a toy model with MWI-style splitting that deals with Bell style experiments in a way that preserves locality

No you can't.
but doesn't require hidden variables (see https://www.mdpi.com/1099-4300/21/1/87/htm ) but I see it as a sort of work in progress rather than a complete interpretation.

They set up a contrast between realism and locality. This is a false contrast, since Bell's theorem has nothing to do with any concept of realism. Bell's concern was to show that the results of quantum mechanics violate the assumption of locality -- there is no other escape. So called "Einsteinian realism" has no role in Bell's argument.

If you think that MWI provides a simple local explanation of the violation of Bell inequalities, then give the argument here -- and not in terms of endless links to nonsense papers.

Bruce

Brent Meeker

unread,
Dec 20, 2021, 8:12:49 PM12/20/21
to everyth...@googlegroups.com


On 12/20/2021 4:52 PM, Jesse Mazer wrote:


On Mon, Dec 20, 2021 at 7:01 PM John Clark <johnk...@gmail.com> wrote:

Brent Meeker meeke...@gmail.com

 Yes, it's empirically supported; So's the Schroedinger equation.  But it's part of the application of the Schroedinger equation.  It's not in the equation itself. 

> I don't know what you mean by that. 
> It's the projection postulate in the Copenhagen interpretation that applies the Born rule.  In MWI it's the Born rule plus some kind of self-locating uncertainty to allow for the probabilistic observations.  So those are things not in the Schroedinger equation.

I don't know how you figure that. It has been mathematically proven that the Born rule is the only way to get probabilities out of Schrodinger's equation such that all the probabilities add up to 1. And Schrodinger says an electron wave can be in any location, and in a camera/electron wave a camera will observe the electron being in every location, and in a Brent Meeker/camera/electron wave there will be a  Brent Meeker for every camera that sees an electron in every location.

> No, you can't observe the Born rule to be true any more (or less) than you can observe Schroedinger's equation to be true.

Nonsense! Every quantum physicist alive believes the Born rule is valid and they use it every day, and the reason they're so confident is because the Born rule has always conform with observations and all empirical tests , so it doesn't need a seal of approval  from a theory for us to think it's true, but a theory may need a seal of approval from the Born Rule to convince us that a theory is true. That's because observation always outranks theory.

But one of the big selling points of the MWI is to give some sort of objective picture of reality in which "measurements" have no distinguished role, but are simply treated using the usual rules of quantum interactions. If you have to say "OK, I believe in the MWI plus Born rule for measurements" with there being no dynamical definition of what qualifies as a measurement, where the moments we call 'measurements' are just something we feed into the theory on a know-it-when-I-see-it basis, then this claim to objectivity is lost and it's not clear what theoretical appeal it has over the Copenhagen interpretation. 

Personally I still lean towards some version of the MWI being true mainly because you can come up with a toy model with MWI-style splitting that deals with Bell style experiments in a way that preserves locality but doesn't require hidden variables (see https://www.mdpi.com/1099-4300/21/1/87/htm ) but I see it as a sort of work in progress rather than a complete interpretation.

I agree.  MWI is useful because it motivated the research into decoherence theory.  But if you just take MWI+probability+basis you can pretty much force the Born rule.  But what does probability or basis mean for the 1e50 interactions taking place all around you that are not "measurements"...or are they?  MWI's whole point is to avoid any distinction between measurement and other interactions.

Brent

Brent Meeker

unread,
Dec 20, 2021, 8:36:12 PM12/20/21
to everyth...@googlegroups.com


On 12/20/2021 5:10 PM, Bruce Kellett wrote:
On Tue, Dec 21, 2021 at 11:53 AM Jesse Mazer <laser...@gmail.com> wrote:
On Mon, Dec 20, 2021 at 7:01 PM John Clark <johnk...@gmail.com> wrote:

Brent Meeker meeke...@gmail.com

 Yes, it's empirically supported; So's the Schroedinger equation.  But it's part of the application of the Schroedinger equation.  It's not in the equation itself. 

> I don't know what you mean by that. 
> It's the projection postulate in the Copenhagen interpretation that applies the Born rule.  In MWI it's the Born rule plus some kind of self-locating uncertainty to allow for the probabilistic observations.  So those are things not in the Schroedinger equation.

I don't know how you figure that. It has been mathematically proven that the Born rule is the only way to get probabilities out of Schrodinger's equation such that all the probabilities add up to 1. And Schrodinger says an electron wave can be in any location, and in a camera/electron wave a camera will observe the electron being in every location, and in a Brent Meeker/camera/electron wave there will be a  Brent Meeker for every camera that sees an electron in every location.

> No, you can't observe the Born rule to be true any more (or less) than you can observe Schroedinger's equation to be true.

Nonsense! Every quantum physicist alive believes the Born rule is valid and they use it every day, and the reason they're so confident is because the Born rule has always conform with observations and all empirical tests , so it doesn't need a seal of approval  from a theory for us to think it's true, but a theory may need a seal of approval from the Born Rule to convince us that a theory is true. That's because observation always outranks theory.

But one of the big selling points of the MWI is to give some sort of objective picture of reality in which "measurements" have no distinguished role, but are simply treated using the usual rules of quantum interactions.

At one time, that might have been a point on which to prefer MWI over Bohr's version of the CI, but that is no longer true. Modern collapse theories do not have to distinguish particular "measurement" events, and do not have to assume a classical superstructure . In modern fGRW, for example, everything can be treated as quantum, and the theory is completely objective.

fGRW has the added advantage that it is an inherently stochastic theory. Probability is treated as a primitive notion that is not based on anything else. MWI struggles with the concept of probability, and while it has to reject a frequentist basis for probability, it cannot really supply anything else. Self-locating uncertainty does not, in itself, serve to define probability. You have to have some notion of a random selection from a set, and that is not available in either the Schrodinger equation or in self-locating uncertainty.

In principle one should be able to empirically distinguish between wave function "collapse" due to GRW or due to decoherence.  It would take extreme isolation to suppress decoherence though.

Brent

Bruce Kellett

unread,
Dec 20, 2021, 8:57:11 PM12/20/21
to Everything List
Sure. GRW collapse is experimentally testable, at least in principle. But MWI is not susceptible to any empirical test.

Bruce

John Clark

unread,
Dec 20, 2021, 10:32:43 PM12/20/21
to 'Brent Meeker' via Everything List
On Mon, Dec 20, 2021 at 7:50 PM Brent Meeker <meeke...@gmail.com> wrote:

> MWI is completely deterministic, including the prediction that all possibilities occur. 

True.
 
> So you have to have some assumption to get probabilities, such that one thing happens and others don't. 

Yes, that is the one assumption you have to make in the MWI, you have to assume that the Schrodinger wave equation means what it says, and in words it says  "The rate of change of a wave function is proportional to the energy of the quantum system and the high energy parts of the wave function evolve rapidly while the low energy parts evolve slowly". It would be expected that more things happen in the rapidly evolving parts then the slowly evolving parts.
 
> MWI finesses this by saying that you observe all possible outcomes...but in other worlds. 

That depends on the meaning of the pronoun "you". In the fast evolving part of the wave function more things are happening but there are also more versions of "you" to see them, and some parts contain no energy at all and thus nothing happens there at all. It is physically impossible for some things to happen so no version of "you" sees it.  

> But the mechanism of this splitting, when and where it happens, is as just as hand wavy as Copenhagen's projection postulate.

No, it's right there in the equation, the thing is that people forget that they are a quantum system too and thus are also part of the Schrodinger wave equation. The equation says nothing about a separation between the observer and the thing that is being observed, that is just pasted  on by every quantum interpretation except for  Many Worlds. MWI is strip down bare bones no nonsense Quantum Mechanics with none of the silly gimmicks tacked on just to make those who dislike the idea that they are not unique feel good.   

>> And Schrodinger says an electron wave can be in any location, and in a camera/electron wave a camera will observe the electron being in every location, and in a Brent Meeker/camera/electron wave there will be a  Brent Meeker for every camera that sees an electron in every location.

> That's like saying every horse in the gate is a possible winner of the Kentucy derby. But that doesn't get you to probabilities without an assumption that one an only one will win.  Everett wants to avoid that assumption...which then takes self-locating uncertainty to make it consistent with probabilistic observations.

Schrodinger equation says high energy parts of the wave evolve swiftly, so there would be more versions of you in those parts than in the low energy parts, so it's more likely that "you" will end up in a higher energy part than a low energy part.  

 >> Every quantum physicist alive believes the Born rule is valid and they use it every day, and the reason they're so confident is because the Born rule has always conform with observations and all empirical tests , so it doesn't need a seal of approval  from a theory for us to think it's true, but a theory may need a seal of approval from the Born Rule to convince us that a theory is true. That's because observation always outranks theory. 

 
> But observation is always finite, while theories claim infinite applicability.  Newton's mechanics is also used everyday, with confidence.  I didn't say theory made it true.  Theory only shows the Born rule is consistent with Hilbert space. 

And if the Born rule had been proven to be inconsistent with Hilbert space physicist would not have gotten rid of the Born rule, instead they would've gotten rid of Hilbert space, because the Born rule would have continued to work regardless of what Hilbert space's opinion of it is.

John K Clark    See what's on my new list at  Extropolis
hsx

Stathis Papaioannou

unread,
Dec 20, 2021, 10:58:48 PM12/20/21
to everyth...@googlegroups.com
On Tue, 21 Dec 2021 at 14:32, John Clark <johnk...@gmail.com> wrote:
On Mon, Dec 20, 2021 at 7:50 PM Brent Meeker <meeke...@gmail.com> wrote:

> MWI is completely deterministic, including the prediction that all possibilities occur. 

True.
 
> So you have to have some assumption to get probabilities, such that one thing happens and others don't. 

Yes, that is the one assumption you have to make in the MWI, you have to assume that the Schrodinger wave equation means what it says, and in words it says  "The rate of change of a wave function is proportional to the energy of the quantum system and the high energy parts of the wave function evolve rapidly while the low energy parts evolve slowly". It would be expected that more things happen in the rapidly evolving parts then the slowly evolving parts.
 
> MWI finesses this by saying that you observe all possible outcomes...but in other worlds. 

That depends on the meaning of the pronoun "you". In the fast evolving part of the wave function more things are happening but there are also more versions of "you" to see them, and some parts contain no energy at all and thus nothing happens there at all. It is physically impossible for some things to happen so no version of "you" sees it.  

But there are events such as the decay of an atom within a half life period that one version of you will see and another version of you will see, which is interpreted as a 1/2 probability of you seeing the atom decay, if you have a normal human brain without telepathic communication with other copies.

> But the mechanism of this splitting, when and where it happens, is as just as hand wavy as Copenhagen's projection postulate.

No, it's right there in the equation, the thing is that people forget that they are a quantum system too and thus are also part of the Schrodinger wave equation. The equation says nothing about a separation between the observer and the thing that is being observed, that is just pasted  on by every quantum interpretation except for  Many Worlds. MWI is strip down bare bones no nonsense Quantum Mechanics with none of the silly gimmicks tacked on just to make those who dislike the idea that they are not unique feel good.   

>> And Schrodinger says an electron wave can be in any location, and in a camera/electron wave a camera will observe the electron being in every location, and in a Brent Meeker/camera/electron wave there will be a  Brent Meeker for every camera that sees an electron in every location.

> That's like saying every horse in the gate is a possible winner of the Kentucy derby. But that doesn't get you to probabilities without an assumption that one an only one will win.  Everett wants to avoid that assumption...which then takes self-locating uncertainty to make it consistent with probabilistic observations.

Schrodinger equation says high energy parts of the wave evolve swiftly, so there would be more versions of you in those parts than in the low energy parts, so it's more likely that "you" will end up in a higher energy part than a low energy part.  

 >> Every quantum physicist alive believes the Born rule is valid and they use it every day, and the reason they're so confident is because the Born rule has always conform with observations and all empirical tests , so it doesn't need a seal of approval  from a theory for us to think it's true, but a theory may need a seal of approval from the Born Rule to convince us that a theory is true. That's because observation always outranks theory. 

 
> But observation is always finite, while theories claim infinite applicability.  Newton's mechanics is also used everyday, with confidence.  I didn't say theory made it true.  Theory only shows the Born rule is consistent with Hilbert space. 

And if the Born rule had been proven to be inconsistent with Hilbert space physicist would not have gotten rid of the Born rule, instead they would've gotten rid of Hilbert space, because the Born rule would have continued to work regardless of what Hilbert space's opinion of it is.

John K Clark    See what's on my new list at  Extropolis
hsx

--
You received this message because you are subscribed to the Google Groups "Everything List" group.
To unsubscribe from this group and stop receiving emails from it, send an email to everything-li...@googlegroups.com.
--
Stathis Papaioannou

Brent Meeker

unread,
Dec 21, 2021, 12:32:47 AM12/21/21
to everyth...@googlegroups.com


On 12/20/2021 7:32 PM, John Clark wrote:
On Mon, Dec 20, 2021 at 7:50 PM Brent Meeker <meeke...@gmail.com> wrote:

> MWI is completely deterministic, including the prediction that all possibilities occur. 

True.
 
> So you have to have some assumption to get probabilities, such that one thing happens and others don't. 

Yes, that is the one assumption you have to make in the MWI, you have to assume that the Schrodinger wave equation means what it says, and in words it says  "The rate of change of a wave function is proportional to the energy of the quantum system and the high energy parts of the wave function evolve rapidly while the low energy parts evolve slowly". It would be expected that more things happen in the rapidly evolving parts then the slowly evolving parts.

Whether the Geiger counter detects five alpha particles in a second or four doesn't depend on some atoms evolving slowly or quickly.


 
> MWI finesses this by saying that you observe all possible outcomes...but in other worlds. 

That depends on the meaning of the pronoun "you". In the fast evolving part of the wave function more things are happening but there are also more versions of "you" to see them, and some parts contain no energy at all and thus nothing happens there at all. It is physically impossible for some things to happen so no version of "you" sees it. 

That's a strange thing to say.  In the last few seconds thousands of cosmic rays shot thru you and you didn't see or detect them in any way.  Yet according Everett they split the world into as many copies because they left traces that could be observed where they passed thru solid objects. 


> But the mechanism of this splitting, when and where it happens, is as just as hand wavy as Copenhagen's projection postulate.

No, it's right there in the equation, the thing is that people forget that they are a quantum system too and thus are also part of the Schrodinger wave equation. The equation says nothing about a separation between the observer and the thing that is being observed, that is just pasted  on by every quantum interpretation except for  Many Worlds. MWI is strip down bare bones no nonsense Quantum Mechanics with none of the silly gimmicks tacked on just to make those who dislike the idea that they are not unique feel good.   

>> And Schrodinger says an electron wave can be in any location, and in a camera/electron wave a camera will observe the electron being in every location, and in a Brent Meeker/camera/electron wave there will be a  Brent Meeker for every camera that sees an electron in every location.

> That's like saying every horse in the gate is a possible winner of the Kentucy derby. But that doesn't get you to probabilities without an assumption that one an only one will win.  Everett wants to avoid that assumption...which then takes self-locating uncertainty to make it consistent with probabilistic observations.

Schrodinger equation says high energy parts of the wave evolve swiftly, so there would be more versions of you in those parts than in the low energy parts, so it's more likely that "you" will end up in a higher energy part than a low energy part.  

 >> Every quantum physicist alive believes the Born rule is valid and they use it every day, and the reason they're so confident is because the Born rule has always conform with observations and all empirical tests , so it doesn't need a seal of approval  from a theory for us to think it's true, but a theory may need a seal of approval from the Born Rule to convince us that a theory is true. That's because observation always outranks theory. 

 
> But observation is always finite, while theories claim infinite applicability.  Newton's mechanics is also used everyday, with confidence.  I didn't say theory made it true.  Theory only shows the Born rule is consistent with Hilbert space. 

And if the Born rule had been proven to be inconsistent with Hilbert space physicist would not have gotten rid of the Born rule, instead they would've gotten rid of Hilbert space, because the Born rule would have continued to work regardless of what Hilbert space's opinion of it is.

Without Hilbert space they'd have no state vector to apply the Born rule.

Brent

Jesse Mazer

unread,
Dec 21, 2021, 12:40:28 AM12/21/21
to everyth...@googlegroups.com
On Mon, Dec 20, 2021 at 8:10 PM Bruce Kellett <bhkel...@gmail.com> wrote:
On Tue, Dec 21, 2021 at 11:53 AM Jesse Mazer <laser...@gmail.com> wrote:
On Mon, Dec 20, 2021 at 7:01 PM John Clark <johnk...@gmail.com> wrote:

Brent Meeker meeke...@gmail.com

 Yes, it's empirically supported; So's the Schroedinger equation.  But it's part of the application of the Schroedinger equation.  It's not in the equation itself. 

> I don't know what you mean by that. 
> It's the projection postulate in the Copenhagen interpretation that applies the Born rule.  In MWI it's the Born rule plus some kind of self-locating uncertainty to allow for the probabilistic observations.  So those are things not in the Schroedinger equation.

I don't know how you figure that. It has been mathematically proven that the Born rule is the only way to get probabilities out of Schrodinger's equation such that all the probabilities add up to 1. And Schrodinger says an electron wave can be in any location, and in a camera/electron wave a camera will observe the electron being in every location, and in a Brent Meeker/camera/electron wave there will be a  Brent Meeker for every camera that sees an electron in every location.

> No, you can't observe the Born rule to be true any more (or less) than you can observe Schroedinger's equation to be true.

Nonsense! Every quantum physicist alive believes the Born rule is valid and they use it every day, and the reason they're so confident is because the Born rule has always conform with observations and all empirical tests , so it doesn't need a seal of approval  from a theory for us to think it's true, but a theory may need a seal of approval from the Born Rule to convince us that a theory is true. That's because observation always outranks theory.

But one of the big selling points of the MWI is to give some sort of objective picture of reality in which "measurements" have no distinguished role, but are simply treated using the usual rules of quantum interactions.

At one time, that might have been a point on which to prefer MWI over Bohr's version of the CI, but that is no longer true. Modern collapse theories do not have to distinguish particular "measurement" events, and do not have to assume a classical superstructure . In modern fGRW, for example, everything can be treated as quantum, and the theory is completely objective.

fGRW has the added advantage that it is an inherently stochastic theory. Probability is treated as a primitive notion that is not based on anything else. MWI struggles with the concept of probability, and while it has to reject a frequentist basis for probability, it cannot really supply anything else. Self-locating uncertainty does not, in itself, serve to define probability. You have to have some notion of a random selection from a set, and that is not available in either the Schrodinger equation or in self-locating uncertainty.

What does fGRW stand for? If it's stochastic, do you mean it's one of those theories that involves stochastic spontaneous collapse? Such theories are usually in principle experimentally distinguishable from QM, would that be true of this theory as well?

 


If you have to say "OK, I believe in the MWI plus Born rule for measurements" with there being no dynamical definition of what qualifies as a measurement, where the moments we call 'measurements' are just something we feed into the theory on a know-it-when-I-see-it basis, then this claim to objectivity is lost and it's not clear what theoretical appeal it has over the Copenhagen interpretation. 

Personally I still lean towards some version of the MWI being true mainly because you can come up with a toy model with MWI-style splitting that deals with Bell style experiments in a way that preserves locality

No you can't.
but doesn't require hidden variables (see https://www.mdpi.com/1099-4300/21/1/87/htm ) but I see it as a sort of work in progress rather than a complete interpretation.

They set up a contrast between realism and locality.

I wasn't linking to the paper for the argument about semantics (there doesn't seem to be any agreed-upon definition of 'realism' distinct from local realism in physics, from what I've seen) but rather for the toy model they provide in section 5 with the experimenters being duplicated when they try to measure the entangled particle. The point is that Alice is locally duplicated when she measures her particle, and Bob is locally duplicated when he measures his, but there is no need for the universe to decide which copy of Bob inhabits the same "world" as a given copy of Alice, or vice versa, until there's been time for signals limited by the speed of light to pass between them (or to a third observer). This is not the sort of "local realist" theory that Bell was trying to refute (one of the implicit assumptions in his derivation was that each spin measurement produces exactly one of two possible outcomes), but the dynamics of such splitting can be perfectly local, and it can still be true that if you randomly select one of the copies of an observer in a Bell type experiment, the probabilities that your randomly selected copy will see various outcomes can be made to match the QM predictions that violate Bell inequalities. 

As I said this can only be shown clearly in a toy model like the one in section 5 of that paper, but a number of physicists including David Deutsch do think that the full MWI would also respect a principle of "local splitting", although even if this can be shown in terms of the quantum formalism we still have the problem of deriving probabilities. The article on the MWI by Lev Vaidman at https://plato.stanford.edu/entries/qm-manyworlds/ discusses work on the notion of local splitting in the MWI:

'Deutsch 2012 claims to provide an alternative vindication of quantum locality using a quantum information framework. This approach started with Deutsch and Hayden 2000 analyzing the flow of quantum information using the Heisenberg picture. After discussions by Rubin 2001 and Deutsch 2002, Hewitt-Horsman and Vedral 2007 analyzed the uniqueness of the physical picture of the information flow. Timpson 2005 and Wallace and Timpson 2007 questioned the locality demonstration in this approach and the meaning of the locality claim was clarified in Deutsch 2012. Rubin 2011 suggested that this approach might provide a simpler route toward generalization of the MWI of quantum mechanics to the MWI of field theory. Recent works Raymond-Robichaud 2020, Kuypers and Deutsch 2021, Bédard 2021a, clarified the meaning of the Deutsch and Hayden proposal as an alternative local MWI which not only lacks action at a distance, but provides a set of local descriptions which completely describes the whole physical Universe. However, there is a complexity price. Bédard 2021b argues that “the descriptor of a single qubit has larger dimensionality than the Schrödinger state of the whole network or of the Universe!”'

Bruce Kellett

unread,
Dec 21, 2021, 1:12:39 AM12/21/21
to Everything List
On Tue, Dec 21, 2021 at 4:40 PM Jesse Mazer <laser...@gmail.com> wrote:
On Mon, Dec 20, 2021 at 8:10 PM Bruce Kellett <bhkel...@gmail.com> wrote:
On Tue, Dec 21, 2021 at 11:53 AM Jesse Mazer <laser...@gmail.com> wrote:

But one of the big selling points of the MWI is to give some sort of objective picture of reality in which "measurements" have no distinguished role, but are simply treated using the usual rules of quantum interactions.

At one time, that might have been a point on which to prefer MWI over Bohr's version of the CI, but that is no longer true. Modern collapse theories do not have to distinguish particular "measurement" events, and do not have to assume a classical superstructure . In modern fGRW, for example, everything can be treated as quantum, and the theory is completely objective.

fGRW has the added advantage that it is an inherently stochastic theory. Probability is treated as a primitive notion that is not based on anything else. MWI struggles with the concept of probability, and while it has to reject a frequentist basis for probability, it cannot really supply anything else. Self-locating uncertainty does not, in itself, serve to define probability. You have to have some notion of a random selection from a set, and that is not available in either the Schrodinger equation or in self-locating uncertainty.

What does fGRW stand for?

It is short for Flash-GRW, in which the random collapse interactions of GRW are replaced by "flashes". The point here is that this formulation is Lorentz invariant and completely relativistic.

If it's stochastic, do you mean it's one of those theories that involves stochastic spontaneous collapse? Such theories are usually in principle experimentally distinguishable from QM, would that be true of this theory as well?

In principle this collapse model is distinguishable from no-collapse models. The experiments to detect this might be outside current capabilities.

If you have to say "OK, I believe in the MWI plus Born rule for measurements" with there being no dynamical definition of what qualifies as a measurement, where the moments we call 'measurements' are just something we feed into the theory on a know-it-when-I-see-it basis, then this claim to objectivity is lost and it's not clear what theoretical appeal it has over the Copenhagen interpretation. 

Personally I still lean towards some version of the MWI being true mainly because you can come up with a toy model with MWI-style splitting that deals with Bell style experiments in a way that preserves locality

No you can't.
but doesn't require hidden variables (see https://www.mdpi.com/1099-4300/21/1/87/htm ) but I see it as a sort of work in progress rather than a complete interpretation.

They set up a contrast between realism and locality.

I wasn't linking to the paper for the argument about semantics (there doesn't seem to be any agreed-upon definition of 'realism' distinct from local realism in physics, from what I've seen) but rather for the toy model they provide in section 5 with the experimenters being duplicated when they try to measure the entangled particle. The point is that Alice is locally duplicated when she measures her particle, and Bob is locally duplicated when he measures his, but there is no need for the universe to decide which copy of Bob inhabits the same "world" as a given copy of Alice, or vice versa, until there's been time for signals limited by the speed of light to pass between them (or to a third observer). This is not the sort of "local realist" theory that Bell was trying to refute (one of the implicit assumptions in his derivation was that each spin measurement produces exactly one of two possible outcomes), but the dynamics of such splitting can be perfectly local, and it can still be true that if you randomly select one of the copies of an observer in a Bell type experiment, the probabilities that your randomly selected copy will see various outcomes can be made to match the QM predictions that violate Bell inequalities.

This seems to be the hand-waving way in which this is usually argued. I was asking for something a little more concrete.

There is a fairly simple argument that shows that many worlds ideas can have no role to play in the violation of the Bell inequalities. In other words, there is an indirect no-go theorem for the idea that MWI makes these experiments completely local.

The argument goes like this. Take Alice and Bob measuring spin states on members of entangled pairs of particles -- they are presumed to be distant from each other, and independent. Alice, say, measures a sequence of particles at random polarizer orientations, randomizing the polarizer angle between measurements. She records her results (up or down) in a lab book. After N such pairs have been measured, her lab book contains a sequence of N 0s or 1s (for up/down), with a record of the relevant polarizer angle for each measurement. If MWI is correct, there are 2^N copies of Alice, each with a lab book containing a similar binary sequence. Over the 2^N copies of Alice, all possible binary sequences are covered. Bob does the same, so he has a lab book with some binary sequence of 0s and 1s (and 2^N copies with different lab books). For each copy of Bob, and each lab book, all N measurements were necessarily made in the same world (because individuals cannot move between worlds).

 After all measurements are complete, Alice and Bob meet and compare their lab books in order to calculate the correlations between results for different relative measurement angles. Once Alice and Bob meet, they are necessarily in the same world. And since they carry their lab books with them, the measurements made in each lab book must all have been made in that same, single, world. The correlations that Alice and Bob calculate are shown to violate the Bell inequality. (That is experimentally verified). But this violation of the inequality takes place in just one world, as has been seen by the above construction. The alternative copies of Alice and Bob also meet to compare results. As before, all these meetings take place in the same worlds as all the relevant measurements were made. Consequently, the many-worlds analysis for each Alice-Bob pair is exactly the same as the single world analysis obtained if collapse is assumed. Many-worlds adds nothing to the analysis, so MWI cannot give any alternative explanation of the correlations. In particular, MWI cannot give a local account.

Bruce

 
As I said this can only be shown clearly in a toy model like the one in section 5 of that paper, but a number of physicists including David Deutsch do think that the full MWI would also respect a principle of "local splitting", although even if this can be shown in terms of the quantum formalism we still have the problem of deriving probabilities. The article on the MWI by Lev Vaidman at https://plato.stanford.edu/entries/qm-manyworlds/ discusses work on the notion of local splitting in the MWI:

'Deutsch 2012 claims to provide an alternative vindication of quantum locality using a quantum information framework. This approach started with Deutsch and Hayden 2000 analyzing the flow of quantum information using the Heisenberg picture. After discussions by Rubin 2001 and Deutsch 2002, Hewitt-Horsman and Vedral 2007 analyzed the uniqueness of the physical picture of the information flow. Timpson 2005 and Wallace and Timpson 2007 questioned the locality demonstration in this approach and the meaning of the locality claim was clarified in Deutsch 2012. Rubin 2011 suggested that this approach might provide a simpler route toward generalization of the MWI of quantum mechanics to the MWI of field theory. Recent works Raymond-Robichaud 2020, Kuypers and Deutsch 2021, Bédard 2021a, clarified the meaning of the Deutsch and Hayden proposal as an alternative local MWI which not only lacks action at a distance, but provides a set of local descriptions which completely describes the whole physical Universe. However, there is a complexity price. Bédard 2021b argues that “the descriptor of a single qubit has larger dimensionality than the Schrödinger state of the whole network or of the Universe!”'

I did suggest that you made the argument yourself rather than giving a long list of references.

B.

John Clark

unread,
Dec 21, 2021, 5:08:33 AM12/21/21
to 'Brent Meeker' via Everything List
On Tue, Dec 21, 2021 at 12:32 AM Brent Meeker <meeke...@gmail.com> wrote:

>> that is the one assumption you have to make in the MWI, you have to assume that the Schrodinger wave equation means what it says, and in words it says  "The rate of change of a wave function is proportional to the energy of the quantum system and the high energy parts of the wave function evolve rapidly while the low energy parts evolve slowly". It would be expected that more things happen in the rapidly evolving parts then the slowly evolving parts.

>
Whether the Geiger counter detects five alpha particles in a second or four doesn't depend on some atoms evolving slowly or quickly.

Yes, but that is in no way inconsistent with what I said in the above, in fact that's the reason that all versions of "you" agree on what the half life of a radioactive element is, although they may disagree on whether a particular atom has decayed or not.  

> MWI finesses this by saying that you observe all possible outcomes...but in other worlds. 

>> That depends on the meaning of the pronoun "you". In the fast evolving part of the wave function more things are happening but there are also more versions of "you" to see them, and some parts contain no energy at all and thus nothing happens there at all. It is physically impossible for some things to happen so no version of "you" sees it. 

> That's a strange thing to say.

Yes it's a very strange thing to say no doubt about it, but there is absolutely positively no way any quantum interpretation that is compatible with observation will EVER be able to make the quantum world not seem strange. When you get down into the quantum realm things just seem weird, but they never become logically paradoxical. The reason things seem so strange to us is that there would've been no Evolutionarily advantage to our hominid ancestors on the African savanna if our minds were constructed in such a way that such things seemed intuitively obvious, so instead evolution made our brains good at other things, like avoiding predators and detecting prey.  
 
  > In the last few seconds thousands of cosmic rays shot thru you and you didn't see or detect them in any way. 

Yes.
 
> Yet according Everett they split the world into as many copies because they left traces that could be observed where they passed thru solid objects. 

Yes. Is there supposed to be a problem with that?

>> And if the Born rule had been proven to be inconsistent with Hilbert space physicist would not have gotten rid of the Born rule, instead they would've gotten rid of Hilbert space, because the Born rule would have continued to work regardless of what Hilbert space's opinion of it is.

> Without Hilbert space they'd have no state vector to apply the Born rule.

And that would be a pity, but physicists would still have the ability to multiply numbers and find their square roots, they were doing such numerical manipulation long before anybody knew anything about Hilbert spaceso they could still use the Born rule. Regardless of what a mathematician might say physicists will never abandon the Born rule as long as it retains its ability to make successful predictions.

John K Clark    See what's on my new list at  Extropolis
q92


 

John Clark

unread,
Dec 21, 2021, 5:52:43 AM12/21/21
to 'Brent Meeker' via Everything List
On Mon, Dec 20, 2021 at 5:28 AM Bruce Kellett <bhkel...@gmail.com> wrote:

>modern collapse theories, such as Flash-GRW, do not have this limitation. There is no observer/observed distinction in such theories, and they can easily accommodate the idea that everything, including the observer, is quantum.

One thing GRW can't accommodate is Special Relativity, so it's inconsistent with observation, so it's not yet a quantum interpretation at all, but Many Worlds had no difficulty in accommodating Special Relativity from day one. Unlike Many Worlds GRW is not deterministic, it adds a random term to Schrodinger's equation that only does 4 things:

1) It makes the new equation inconsistent with special relativity and thus observation. 
2) It makes an equation that was already very difficult to solve even more difficult.
3) It makes Schrodinger's equation become nondeterministic. 
4) It gets rid of those Many Worlds that so many people hate and fear.

Maybe someday GRW Will do better but that would require a complete rewrite, and the prospects are not looking good: 


John K Clark    See what's on my new list at  Extropolis
wrd

Bruce Kellett

unread,
Dec 21, 2021, 6:11:05 AM12/21/21
to Everything List
Flash-GRW is Lorentz invariant and completely relativistic because it is based on light cone physics. Getting rid of the determinism of the Schrodinger equation is a good thing if you want a theory that is going to predict probabilities.

Bruce

John Clark

unread,
Dec 21, 2021, 6:29:26 AM12/21/21
to 'Brent Meeker' via Everything List
On Tue, Dec 21, 2021 at 6:11 AM Bruce Kellett <bhkel...@gmail.com> wrote:

>> Maybe someday GRW Will do better but that would require a complete rewrite, and the prospects are not looking good: 



> Flash-GRW is Lorentz invariant and completely relativistic

You should submit a paper to Physical Review to compete with the one they just published. 
 
> Getting rid of the determinism of the Schrodinger equation is a good thing if you want a theory that is going to predict probabilities.

That doesn't follow because there is a difference between ontology and epistemology.

John K Clark    See what's on my new list at  Extropolis
xx2
 

smitra

unread,
Dec 21, 2021, 8:53:51 AM12/21/21
to everyth...@googlegroups.com
It is the violation of the Bell inequality in each world that is the
evidence of the existence of the other worlds. The problem is with
comparing with collapse hypothesis and then saying that there is no
difference. But the whole problem is that when Alice makes her
measurement that she gains some amount of information about what Bob is
going to find, even though they are spacelike separated. In the MWI
there is no such mysterious gain of information due to the correlation
being caused by common cause when the entangled pair is created.

Saibal



John Clark

unread,
Dec 21, 2021, 12:01:30 PM12/21/21
to 'Brent Meeker' via Everything List
On Mon, Dec 20, 2021 at 10:58 PM Stathis Papaioannou <stat...@gmail.com> wrote:

>but there are events such as the decay of an atom within a half life period that one version of you will see and another version of you will see, which is interpreted as a 1/2 probability of you seeing the atom decay, if you have a normal human brain without telepathic communication with other copies.
 
All versions of "you" that live in worlds that have the same fundamental laws of physics (and those that don't would be so different they probably wouldn't deserve to be called "you") would agree that Neptunium 240 has a half-life of one hour, in other words that mode of decay would be the most common and most of "you" in the multi-verse would see an atom of Neptunium decay at around the one hour mark. But most does not mean all and if we're talking about one particular Neptunium 240 atom a minority of "you" will not see it decay after 5 hours even though you know it's half life is only one hour, and a very tiny minority will not see it decay even after 5 million years, and another very tiny minority of "you" will see it decay after only 5 nanoseconds.

You may ask, how different can "you" be before it no longer deserves the right to be called "you"? I admit that limit is somewhat arbitrary, but the important point is that whatever limit you choose, as long as it's consistent, it makes no difference if high precision is demanded for something to be called "you" or if extreame sloppiness can be tolerated, either way it will still remain true that there will be more "yous" near the center of the Bell Curve than at the trailing edges.


John K Clark    See what's on my new list at  Extropolis
emc

Jesse Mazer

unread,
Dec 21, 2021, 12:45:51 PM12/21/21
to everyth...@googlegroups.com
On Tue, Dec 21, 2021 at 1:12 AM Bruce Kellett <bhkel...@gmail.com> wrote:
On Tue, Dec 21, 2021 at 4:40 PM Jesse Mazer <laser...@gmail.com> wrote:
On Mon, Dec 20, 2021 at 8:10 PM Bruce Kellett <bhkel...@gmail.com> wrote:
On Tue, Dec 21, 2021 at 11:53 AM Jesse Mazer <laser...@gmail.com> wrote:

But one of the big selling points of the MWI is to give some sort of objective picture of reality in which "measurements" have no distinguished role, but are simply treated using the usual rules of quantum interactions.

At one time, that might have been a point on which to prefer MWI over Bohr's version of the CI, but that is no longer true. Modern collapse theories do not have to distinguish particular "measurement" events, and do not have to assume a classical superstructure . In modern fGRW, for example, everything can be treated as quantum, and the theory is completely objective.

fGRW has the added advantage that it is an inherently stochastic theory. Probability is treated as a primitive notion that is not based on anything else. MWI struggles with the concept of probability, and while it has to reject a frequentist basis for probability, it cannot really supply anything else. Self-locating uncertainty does not, in itself, serve to define probability. You have to have some notion of a random selection from a set, and that is not available in either the Schrodinger equation or in self-locating uncertainty.

What does fGRW stand for?

It is short for Flash-GRW, in which the random collapse interactions of GRW are replaced by "flashes". The point here is that this formulation is Lorentz invariant and completely relativistic.

I assume the flashes are collapses to eigenstates, with probabilities given by the Born rule, even if these collapses are not necessarily caused by interactions? If so, what factors affect the probability a collapse happens at any given moment? Does it depend on the mass of the entangled system (thus becoming more likely as the system becomes entangled with its environment), as in Penrose's suggestion?
 

If it's stochastic, do you mean it's one of those theories that involves stochastic spontaneous collapse? Such theories are usually in principle experimentally distinguishable from QM, would that be true of this theory as well?

In principle this collapse model is distinguishable from no-collapse models. The experiments to detect this might be outside current capabilities.

If you have to say "OK, I believe in the MWI plus Born rule for measurements" with there being no dynamical definition of what qualifies as a measurement, where the moments we call 'measurements' are just something we feed into the theory on a know-it-when-I-see-it basis, then this claim to objectivity is lost and it's not clear what theoretical appeal it has over the Copenhagen interpretation. 

Personally I still lean towards some version of the MWI being true mainly because you can come up with a toy model with MWI-style splitting that deals with Bell style experiments in a way that preserves locality

No you can't.
but doesn't require hidden variables (see https://www.mdpi.com/1099-4300/21/1/87/htm ) but I see it as a sort of work in progress rather than a complete interpretation.

They set up a contrast between realism and locality.

I wasn't linking to the paper for the argument about semantics (there doesn't seem to be any agreed-upon definition of 'realism' distinct from local realism in physics, from what I've seen) but rather for the toy model they provide in section 5 with the experimenters being duplicated when they try to measure the entangled particle. The point is that Alice is locally duplicated when she measures her particle, and Bob is locally duplicated when he measures his, but there is no need for the universe to decide which copy of Bob inhabits the same "world" as a given copy of Alice, or vice versa, until there's been time for signals limited by the speed of light to pass between them (or to a third observer). This is not the sort of "local realist" theory that Bell was trying to refute (one of the implicit assumptions in his derivation was that each spin measurement produces exactly one of two possible outcomes), but the dynamics of such splitting can be perfectly local, and it can still be true that if you randomly select one of the copies of an observer in a Bell type experiment, the probabilities that your randomly selected copy will see various outcomes can be made to match the QM predictions that violate Bell inequalities.

This seems to be the hand-waving way in which this is usually argued. I was asking for something a little more concrete.

There is a fairly simple argument that shows that many worlds ideas can have no role to play in the violation of the Bell inequalities. In other words, there is an indirect no-go theorem for the idea that MWI makes these experiments completely local.

The argument goes like this. Take Alice and Bob measuring spin states on members of entangled pairs of particles -- they are presumed to be distant from each other, and independent. Alice, say, measures a sequence of particles at random polarizer orientations, randomizing the polarizer angle between measurements. She records her results (up or down) in a lab book. After N such pairs have been measured, her lab book contains a sequence of N 0s or 1s (for up/down), with a record of the relevant polarizer angle for each measurement. If MWI is correct, there are 2^N copies of Alice, each with a lab book containing a similar binary sequence. Over the 2^N copies of Alice, all possible binary sequences are covered. Bob does the same, so he has a lab book with some binary sequence of 0s and 1s (and 2^N copies with different lab books). For each copy of Bob, and each lab book, all N measurements were necessarily made in the same world (because individuals cannot move between worlds).

 After all measurements are complete, Alice and Bob meet and compare their lab books in order to calculate the correlations between results for different relative measurement angles. Once Alice and Bob meet, they are necessarily in the same world. And since they carry their lab books with them, the measurements made in each lab book must all have been made in that same, single, world. The correlations that Alice and Bob calculate are shown to violate the Bell inequality. (That is experimentally verified). But this violation of the inequality takes place in just one world, as has been seen by the above construction. The alternative copies of Alice and Bob also meet to compare results. As before, all these meetings take place in the same worlds as all the relevant measurements were made. Consequently, the many-worlds analysis for each Alice-Bob pair is exactly the same as the single world analysis obtained if collapse is assumed. Many-worlds adds nothing to the analysis, so MWI cannot give any alternative explanation of the correlations. In particular, MWI cannot give a local account.

You seem to be assuming one copy for each distinct measurement outcome, and given that assumption it's true you can't reproduce the statistics of arbitrary Bell type experiments, but if you are allowed to assume *multiple* copies for any given outcome it can be made to work. For example, you could simply assume a huge (approaching infinity) population of copies at the start, and then in a given spin measurement, half the copies see spin-up and half see spin-down--I believe the many-minds interpretation discussed at https://plato.stanford.edu/entries/qm-everett/#ManyMind would work this way. But if we just want to construct a toy model, it may be simpler to assume that each new measurement splits each member of a finite population of copies into further copies. For example, let's say that each measurement splits each existing copy into 8 copies, 4 seeing the result spin-up (which we can denote '1') on that trial, and 4 seeing the result spin-down (which we can denote '0'). So if we start with a single version of Bob who measures 2 particles in a row, there will then be 8^2 = 64 copies of Bob, with 1/4 of them (i.e. 16 copies) recording the result 00, 1/4 recording the result 01, 1/4 recording 10, and 1/4 recording 11. And exactly the same local splitting will occur for Alice when she measures the two entangled twins of the particles Bob measured.

One commonly discussed Bell type experiment allows each experimenter to choose between 3 detector settings on each trial, with the quantum prediction being that on any trial where they both chose the same detector setting they both get the same result with probability 1 (or opposite results with probability 1 depending on the type of particle and the experimental setup, but let's say same results for the sake of simplicity), whereas on any trial where they chose two different detector settings, they have only a 1/4 probability of getting the same result. Bell's theorem would say that given the first condition about the *same* detector settings, we should conclude that when they choose *different* detector settings, any local realist theory would predict they should get the same result at least 1/3 of the time, so the quantum prediction of 1/4 is incompatible with local realism. But as I noted before, it's an implicit condition of "local realism" that each measurement gives a single unique outcome--if we instead have a local model with local copying as I described above, can we subsequently match up the copies of Bob with the copies of Alice in a one-to-one way, such that in a randomly selected matched pair, the probability they saw the same result on any trial with different detector settings was only 1/4?

The answer is yes, and the matching rule is fairly straightforward. Assume for the sake of the argument that both Bob and Alice chose their sequence of detector settings before receiving any particles (and so before they started to get copies), so that in the sequence of two measurements I discussed above, all copies had the same series of detector settings, and can only differ in what results they observed with those settings. Further assume that Alice and Bob chose different detector settings for each measurement, so we want to match up copies in a way that ensures that with a randomly selected matched pair, there is a 1/4 chance they got the same result on the first measurement, and likewise a 1/4 chance they got the same result on the second measurement (and likewise a 3/4 chance they got a different result on each measurement). So, we can match them like this:

--Of the 16 Bobs that got 00, (1/4)*(1/4) = 1/16 are matched with an Alice that got 00, (1/4)*(3/4) = 3/16 are matched with an Alice that got 01, (3/4)*(1/4) = 3/16 are matched with an Alice that got 10, and (3/4)*(3/4) = 9/16 are matched with an Alice that got 11.

--Of the 16 Bobs that got 01, (1/4)*(3/4) = 3/16 are matched with an Alice that got 00, (1/4)*(1/4) = 1/16 are matched with an Alice that got 01, (3/4)*(3/4) = 9/16 are matched with an Alice that got 10, and (3/4)*(1/4)=3/16 are matched with an Alice that got 11.

--Of the 16 Bobs that got 10, (3/4)*(1/4) = 3/16 are matched with an Alice that got 00, (3/4)*(3/4) = 9/16 are matched with an Alice that got 01, (1/4)*(1/4) = 1/16 are matched with an Alice that got 10, and (1/4)*(3/4) = 3/16 are matched with an Alice that got 11.

--Of the 16 Bobs that got 11, (3/4)*(3/4) = 9/16 are matched with an Alice that got 00, (3/4)*(1/4) = 3/16 are matched with an Alice that got 01, (1/4)*(3/4) = 3/16 are matched with an Alice that got 10, and (1/4)*(1/4) = 1/16 are matched with an Alice that got 11.

If you add up the numbers for Alice, to do this matching you need 1 + 3 + 3 + 9 = 16 Alice-copies that get 00, 3 + 1 + 9 + 3 = 16 Alices that get 01, 3 + 9 + 1 + 3 = 16 Alices that get 10, and 9 + 3 + 3 + 1 = 16 Alices that get 11. So, this works out perfectly with the local splitting rule that says when each performs two measurements, each is split into 64 copies with 16 seeing 00, 16 seeing 01, 16 seeing 10, and 16 seeing 11. And the matching rule above was constructed so that if you pick a matched pair at random, on each of the two trials there is a 1/4 chance both had the same result and a 3/4 chance they had different results.

In this case I only looked at a pair of measurements where they chose different detector settings on both, what if we had a mixed series where they both chose the same detector settings on some trials, different detector settings on others? For example let's say they each made 4 measurements, and on the first and last they both chose the same detector setting, while on the middle two they chose different detector settings. Then we will have 8^4 = 4096 copies of Bob, of whom 1/16 = 256 got results 0000, 256 got results 0001, and so forth, and likewise for Alice. So now imagine our matching algorithm first sorting all the Alices and all the Bobs into four "piles" that each have 1024 Bobs and 1024 Alices--in the first pile we will have Bob-copies and Alice-copies that got the result 0 on the first measurement and 0 on the fourth measurement, in the second pile those that got 0 on the first measurement and 1 on the fourth measurement, in the third pile those that got 1 on the first measurement and 0 on the fourth measurement, and in the fourth pile those that got 1 on the first measurement and 1 on the fourth measurement. If we now restrict ourselves to only matching copies of Bob and Alice that are from the same pile, we guarantee that all matched pairs get the same result for the first and fourth measurements, when they were both using the same detector settings. 

And now if we look at a given pile and concentrate on what each copy got on the second and third measurement (when their detector settings were different), we see that 1/4 of the Bobs in that pile got 00 for their 2nd and 3rd measurements, 1/4 got 01, 1/4 got 10, and 1/4 got 11, and likewise for the Alice copies in that pile (since each pile had 1024 Bobs and 1024 Alices, there will be 256 Bobs and 256 Alices in a pile that have each of the 4 possible middle two measurement results). So the within-pile matching of Alice-copies to Bob-copies can proceed the same way as before--for example, of the 256 Bob-copies in a pile that got 00 for their middle measurements, 1/16 are matched with an Alice that got 00 for her middle measurements, 3/16 are matched with an Alice that got 01, 3/16 are matched with an Alice that got 10, and 9/16 are matched with an Alice that got 11.

So, this local-copying-and-later-matching rule does guarantee that if you choose a matched pair at random (with all the pairs equally likely), there's a probability 1 that they got the same result on each trial where they both chose the same detector setting, and a probability 1/4 they got the same result on each trial where they chose different detector settings. It's easy to generalize this rule to arbitrary sequences of N measurements with arbitrary combinations of the same detector setting on some trials, and different detector settings on others.



 
As I said this can only be shown clearly in a toy model like the one in section 5 of that paper, but a number of physicists including David Deutsch do think that the full MWI would also respect a principle of "local splitting", although even if this can be shown in terms of the quantum formalism we still have the problem of deriving probabilities. The article on the MWI by Lev Vaidman at https://plato.stanford.edu/entries/qm-manyworlds/ discusses work on the notion of local splitting in the MWI:

'Deutsch 2012 claims to provide an alternative vindication of quantum locality using a quantum information framework. This approach started with Deutsch and Hayden 2000 analyzing the flow of quantum information using the Heisenberg picture. After discussions by Rubin 2001 and Deutsch 2002, Hewitt-Horsman and Vedral 2007 analyzed the uniqueness of the physical picture of the information flow. Timpson 2005 and Wallace and Timpson 2007 questioned the locality demonstration in this approach and the meaning of the locality claim was clarified in Deutsch 2012. Rubin 2011 suggested that this approach might provide a simpler route toward generalization of the MWI of quantum mechanics to the MWI of field theory. Recent works Raymond-Robichaud 2020, Kuypers and Deutsch 2021, Bédard 2021a, clarified the meaning of the Deutsch and Hayden proposal as an alternative local MWI which not only lacks action at a distance, but provides a set of local descriptions which completely describes the whole physical Universe. However, there is a complexity price. Bédard 2021b argues that “the descriptor of a single qubit has larger dimensionality than the Schrödinger state of the whole network or of the Universe!”'

I did suggest that you made the argument yourself rather than giving a long list of references.

B.

--
You received this message because you are subscribed to the Google Groups "Everything List" group.
To unsubscribe from this group and stop receiving emails from it, send an email to everything-li...@googlegroups.com.

Stathis Papaioannou

unread,
Dec 21, 2021, 3:34:44 PM12/21/21
to everyth...@googlegroups.com
But you have said in the past, with regard to copying experiments, that there is a 100% chance that you (i.e. John Clark) will see both outcomes.
--
Stathis Papaioannou

John Clark

unread,
Dec 21, 2021, 3:50:42 PM12/21/21
to 'Brent Meeker' via Everything List
On Tue, Dec 21, 2021 at 3:34 PM Stathis Papaioannou <stat...@gmail.com> wrote:

>> You may ask, how different can "you" be before it no longer deserves the right to be called "you"? I admit that limit is somewhat arbitrary, but the important point is that whatever limit you choose, as long as it's consistent, it makes no difference if high precision is demanded for something to be called "you" or if extreame sloppiness can be tolerated, either way it will still remain true that there will be more "yous" near the center of the Bell Curve than at the trailing edges.

> But you have said in the past, with regard to copying experiments, that there is a 100% chance that you (i.e. John Clark) will see both outcomes.

Yes, John Clark did say that and sees no reason to retract it because all the John Clark's have as equal a right to that name as the John Clark that is writing this email, even those John Clark's that have observed rare events at the outer edges of the Bell Curve.
John K Clark    See what's on my new list at  Extropolis
bew

Stathis Papaioannou

unread,
Dec 21, 2021, 4:47:29 PM12/21/21
to everyth...@googlegroups.com
So given that John Clark has a 100% probability of seeing each outcome, to whom do the other probabilities apply?


--
Stathis Papaioannou

Bruce Kellett

unread,
Dec 21, 2021, 4:48:57 PM12/21/21
to Everything List
Huh?????

The problem is with
comparing with collapse hypothesis and then saying that there is no
difference.

If there is no difference, where is the problem?

But the whole problem is that when Alice makes her
measurement that she gains some amount of information about what Bob is
going to find, even though they are spacelike separated.

In general, that is not true. When both Alice and Bob set their polarizers randomly while the particles are in flight, the fact that Alice might get |up> tells her nothing about what Bob will get at some randomly different polarizer orientation. You seem to be stuck with thinking in terms of parallel polarizer orientations.


In the MWI
there is no such mysterious gain of information due to the correlation
being caused by common cause when the entangled pair is created

Rubbish. If there were a common cause, then that would have to depend on the final polarizer orientations. And those are not known at the time of creation of the entangled pair. You are, then, back with some non-local influence (or retro-causation).

Bruce

John Clark

unread,
Dec 21, 2021, 5:02:35 PM12/21/21
to 'Brent Meeker' via Everything List
On Tue, Dec 21, 2021 at 4:47 PM Stathis Papaioannou <stat...@gmail.com> wrote:

>>> But you have said in the past, with regard to copying experiments, that there is a 100% chance that you (i.e. John Clark) will see both outcomes.

>> Yes, John Clark did say that and sees no reason to retract it because all the John Clark's have as equal a right to that name as the John Clark that is writing this email, even those John Clark's that have observed rare events at the outer edges of the Bell Curve.

> So given that John Clark has a 100% probability of seeing each outcome, to whom do the other probabilities apply?

This may or may not be the answer because I'm not sure I understand your question, but John Clark will either see event X or see event Y, so there is a 100% chance John Clark will see event X and only event X,  and a 100% chance John Clark will see event Y and only event Y. And the same would be true for people with other names. 
John K Clark    See what's on my new list at  Extropolis
umu

Stathis Papaioannou

unread,
Dec 21, 2021, 5:15:20 PM12/21/21
to everyth...@googlegroups.com
So if you say that there is a 50% chance the atom will decay in the next hour what does that mean, given that there is also a 100% chance the atom will decay in the next hour under MWI?


--
Stathis Papaioannou

John Clark

unread,
Dec 21, 2021, 5:48:51 PM12/21/21
to 'Brent Meeker' via Everything List
On Tue, Dec 21, 2021 at 5:15 PM Stathis Papaioannou <stat...@gmail.com> wrote:

> So if you say that there is a 50% chance the atom will decay in the next hour what does that mean, given that there is also a 100% chance the atom will decay in the next hour under MWI?

By utilizing a complex set of observations it is actually possible to delay the radioactive decay of an atom indefinitely, for practical reasons when this experiment is actually performed the delay is small but statistically significant. This is how Many Worlds would explain that. Suppose an atom has a halflife of one second, the universe splits and so do I after one second.  In one universe the atom decays and in the other it doesn't. In the universe where it didn't decay after another second the universe splits again, and again in one universe it decays but in the other it has not, it survived for 2 full seconds. So there will be a version of me that observes this atom with a one second half life surviving for 3 seconds, and 4 seconds, and 5 years, and 6 centuries, and you name it. By utilizing a series of increasingly complex and difficult procedures in the lab it is possible for the lab to be in the tiny minority of universes that contains observers that see the atom surviving for an arbitrary length of time. But the longer the time and the more atoms involved the more difficult the procedures become and soon becomes ridiculously impractical.

John K Clark    See what's on my new list at  Extropolis
ww3

umu

 





 



--
Stathis Papaioannou

--
You received this message because you are subscribed to the Google Groups "Everything List" group.
To unsubscribe from this group and stop receiving emails from it, send an email to everything-li...@googlegroups.com.

smitra

unread,
Dec 22, 2021, 6:12:06 AM12/22/21
to everyth...@googlegroups.com
Violation of Bell's inequalities rules out local hidden variables. This
means that in some arbitrary quantum experiment where there is more than
one outcome, the information about the measurement result was not
already present locally before the measurement was carried out. So, when
measuring the z-component of a spin 1/2 particle polarized in the
x-direction, the measurement outcome will produce new information out of
thin air, unless, of course, one assumes that the true physical state
contains both outcomes.

>
>> The problem is with
>> comparing with collapse hypothesis and then saying that there is no
>> difference.
>
> If there is no difference, where is the problem?

One needs to invoke a new physical mechanism to explain the collapse.
Since this is then not motivated by experimental results showing that
e.g. systems decohere faster than standard QM would predict, one is then
invoking new physics purely because of a philosophical dislike of a
theory that doesn't need that new physics.
>
>> But the whole problem is that when Alice makes her
>> measurement that she gains some amount of information about what Bob
>> is
>> going to find, even though they are spacelike separated.
>
> In general, that is not true. When both Alice and Bob set their
> polarizers randomly while the particles are in flight, the fact that
> Alice might get |up> tells her nothing about what Bob will get at some
> randomly different polarizer orientation. You seem to be stuck with
> thinking in terms of parallel polarizer orientations.

It's not true only when the polarizers are orthogonal. Whenever the
polarizers are not orthogonal, Alice will gain some amount of
information about what Bob will find given the result of her
measurement. For Bob, the probability of finding up or down are always
1/2, but after Alice makes her measurement, the conditional probability
of what Bob will find, given her measurement result will not be equal to
1/2 for both outcomes if her polarizer was not orthogonal to that of
Bob, so Alice will have gained information about Bob's measurement
result.

>
>> In the MWI
>> there is no such mysterious gain of information due to the
>> correlation
>> being caused by common cause when the entangled pair is created
>
> Rubbish. If there were a common cause, then that would have to depend
> on the final polarizer orientations. And those are not known at the
> time of creation of the entangled pair. You are, then, back with some
> non-local influence (or retro-causation).

The setting of the polarizers will be the result of some physical
process. Whatever you specify for that process should be included in the
analysis of the problem. But when you do so, it's inevitable that in an
MWI analysis, there is not going to be any nonlocal effect other than
trivial common cause effects.

Saibal

Stathis Papaioannou

unread,
Dec 22, 2021, 2:58:39 PM12/22/21
to everyth...@googlegroups.com
On Wed, 22 Dec 2021 at 09:48, John Clark <johnk...@gmail.com> wrote:
On Tue, Dec 21, 2021 at 5:15 PM Stathis Papaioannou <stat...@gmail.com> wrote:

> So if you say that there is a 50% chance the atom will decay in the next hour what does that mean, given that there is also a 100% chance the atom will decay in the next hour under MWI?

By utilizing a complex set of observations it is actually possible to delay the radioactive decay of an atom indefinitely, for practical reasons when this experiment is actually performed the delay is small but statistically significant. This is how Many Worlds would explain that. Suppose an atom has a halflife of one second, the universe splits and so do I after one second.  In one universe the atom decays and in the other it doesn't. In the universe where it didn't decay after another second the universe splits again, and again in one universe it decays but in the other it has not, it survived for 2 full seconds. So there will be a version of me that observes this atom with a one second half life surviving for 3 seconds, and 4 seconds, and 5 years, and 6 centuries, and you name it. By utilizing a series of increasingly complex and difficult procedures in the lab it is possible for the lab to be in the tiny minority of universes that contains observers that see the atom surviving for an arbitrary length of time. But the longer the time and the more atoms involved the more difficult the procedures become and soon becomes ridiculously impractical.

I understand that, but you have previously claimed about thought experiments involving duplication that there is no sense in saying that the subject can expect to see a particular outcome with a particular probability, because he will certainly see all outcomes.
--
Stathis Papaioannou

John Clark

unread,
Dec 22, 2021, 4:15:53 PM12/22/21
to 'Brent Meeker' via Everything List
In the matter duplicating machine thought experiment it is meaningless to ask "what is the probability that you will see city X" because the meaning of the personal pronoun "you" is ambiguous; after the experiment is concluded 2 people in the same world who can talk to each other and to the experimenter both claim to be "you" and one claims he saw city X and the other claims he saw city Y. So the question "what city did you end up seeing?" has no answer because the personal pronoun is ambiguous. By contrast there is no ambiguity at all in the Many Worlds case, "you" is the only person named Stathis Papaioannou that is observable, and I will believe the only Stathis Papaioannou that I can see when he tells me what city he ended up in. But when that same question is asked in the matter duplicating case 2 people with an equally strong claim to be Stathis Papaioannou give contradictory answers to my question, so in the matter duplicating machine case "what is the probability that you will see city X ?" is not a question at all, it's just a string of ASCII characters with a question mark at the end.

John K Clark    See what's on my new list at  Extropolis
am0



 

Stathis Papaioannou

unread,
Dec 22, 2021, 4:46:02 PM12/22/21
to everyth...@googlegroups.com
It's logically possible that we might find a way to meet our copies in other worlds, or that duplication experiments could be set up so that the copies in the same world could never meet.


--
Stathis Papaioannou

Bruce Kellett

unread,
Dec 22, 2021, 4:54:24 PM12/22/21
to Everything List
On Wed, Dec 22, 2021 at 10:12 PM smitra <smi...@zonnet.nl> wrote:
On 21-12-2021 22:48, Bruce Kellett wrote:
>
> In general, that is not true. When both Alice and Bob set their
> polarizers randomly while the particles are in flight, the fact that
> Alice might get |up> tells her nothing about what Bob will get at some
> randomly different polarizer orientation. You seem to be stuck with
> thinking in terms of parallel polarizer orientations.

It's not true only when the polarizers are orthogonal. Whenever the
polarizers are not orthogonal, Alice will gain some amount of
information about what Bob will find given the result of her
measurement. For Bob, the probability of finding up or down are always
1/2, but after Alice makes her measurement, the conditional probability
of what Bob will find, given her measurement result will not be equal to
1/2 for both outcomes if her polarizer was not orthogonal to that of
Bob, so Alice will have gained information about Bob's measurement
result.

The conditional probability you refer to is defined only non-locally.

>> In the MWI
>> there is no such mysterious gain of information due to the correlation
>> being caused by common cause when the entangled pair is created
>
> Rubbish. If there were a common cause, then that would have to depend
> on the final polarizer orientations. And those are not known at the
> time of creation of the entangled pair. You are, then, back with some
> non-local influence (or retro-causation).

The setting of the polarizers will be the result of some physical
process. Whatever you specify for that process should be included in the
analysis of the problem. But when you do so, it's inevitable that in an
MWI analysis, there is not going to be any nonlocal effect other than
trivial common cause effects.

I see. So in desperation you resort to the superdeterminism escape. MWI is not necessary for the understanding of the correlations of entangled particles, as my simple example shows. In an actual experiment, the analysis is identical in many-worlds and collapse models. The additional worlds in MWI add nothing to the explanation. They are, therefore, otiose, and MWI can be discarded.

Bruce

John Clark

unread,
Dec 22, 2021, 5:12:54 PM12/22/21
to 'Brent Meeker' via Everything List
On Wed, Dec 22, 2021 at 4:46 PM Stathis Papaioannou <stat...@gmail.com> wrote:

In the matter duplicating machine thought experiment it is meaningless to ask "what is the probability that you will see city X" because the meaning of the personal pronoun "you" is ambiguous; after the experiment is concluded 2 people in the same world who can talk to each other and to the experimenter both claim to be "you" and one claims he saw city X and the other claims he saw city Y. So the question "what city did you end up seeing?" has no answer because the personal pronoun is ambiguous. By contrast there is no ambiguity at all in the Many Worlds case, "you" is the only person named Stathis Papaioannou that is observable, and I will believe the only Stathis Papaioannou that I can see when he tells me what city he ended up in. But when that same question is asked in the matter duplicating case 2 people with an equally strong claim to be Stathis Papaioannou give contradictory answers to my question, so in the matter duplicating machine case "what is the probability that you will see city X ?" is not a question at all, it's just a string of ASCII characters with a question mark at the end.

> It's logically possible that we might find a way to meet our copies in other worlds, or that duplication experiments could be set up so that the copies in the same world could never meet.

If that ever becomes possible and common (like something out of Rick and Morty) then English and all other human languages are going to need radical revisions, especially in the way they use personal pronouns.  

> or that duplication experiments could be set up so that the copies in the same world could never meet.

After the experiment is completed the experimenter will HAVE TO  communicate with BOTH of them, otherwise it's not an experiment at all and would return zero results.  

John K Clark    See what's on my new list at  Extropolis
ram


Stathis Papaioannou

unread,
Dec 22, 2021, 5:25:16 PM12/22/21
to everyth...@googlegroups.com
It's a thought experiment. You are duplicated in two separate places, A and B, and the two copies can never meet, what is your expectation of finding yourself in A or B? This is equivalent to what happens when the world split according to MWI.

--
Stathis Papaioannou

Bruce Kellett

unread,
Dec 22, 2021, 5:25:25 PM12/22/21
to Everything List
On Wed, Dec 22, 2021 at 10:12 PM smitra <smi...@zonnet.nl> wrote:
On 21-12-2021 22:48, Bruce Kellett wrote:
> On Wed, Dec 22, 2021 at 12:53 AM smitra <smi...@zonnet.nl> wrote:

>> The problem is with
>> comparing with collapse hypothesis and then saying that there is no
>> difference.
>
> If there is no difference, where is the problem?

One needs to invoke a new physical mechanism to explain the collapse.
Since this is then not motivated by experimental results showing that
e.g. systems decohere faster than standard QM would predict, one is then
invoking new physics purely because of a philosophical dislike of a
theory that doesn't need that new physics.

That is a purely philosophical objection!

At least the collapse hypothesis is subject to experimental test. Whereas the many-worlds hypothesis is beyond any conceivable experimental test. Decoherence is not unique to MWI -- it happens in any quantum model.

Bruce

John Clark

unread,
Dec 22, 2021, 5:57:17 PM12/22/21
to 'Brent Meeker' via Everything List
On Wed, Dec 22, 2021 at 5:25 PM Stathis Papaioannou <stat...@gmail.com> wrote:\
> It's a thought experiment. You are duplicated in two separate places, A and B, and the two copies can never meet, what is your expectation of finding yourself in A or B?
 
No. It's not a thought experiment, it's not an experiment of any sort because even after it's all completed nobody has learned anything because nobody has any idea who saw city A and who saw city B;  "you" has been duplicated and thus "you" saw A and "you" saw B, and the question asked is "what one and only one city will "you" end up seeing after "you" is duplicated and becomes two?" has no answer because it is nonsensical .

> This is equivalent to what happens when the world split according to MWI.

No it is not equivalent. In the Many Worlds case the two people are clearly labeled, one of them is observable and the other is not.  In the people duplicating machine case the only difference between the two people is one saw A and one saw B, so the only possible answer to the question "which one saw A" is "the one that saw A" because that's the only label there is to differentiate the two people. 

John K Clark    See what's on my new list at  Extropolis
q96


 

Jesse Mazer

unread,
Dec 22, 2021, 6:01:21 PM12/22/21
to everyth...@googlegroups.com
I don't think Saibal was referring to superdeterminism? Or are you suggesting the MWI version of locality involves superdeterminism? If so that's wrong, superdeterminism involves some special constraint on initial conditions such that variables associated with the entangled particles (hidden or non-hidden) at the moment they are sent out in opposite directions are correlated in advance with the future choices of detector settings by the experimenters.

MWI is not necessary for the understanding of the correlations of entangled particles, as my simple example shows. In an actual experiment, the analysis is identical in many-worlds and collapse models. The additional worlds in MWI add nothing to the explanation.

They allow it to be local without superdeterminism, because the "matching" of local worlds can be done at a point in spacetime that has both experimenter's measurements in its past light cone, I gave you a toy model demonstrating how this can work in my post at https://www.mail-archive.com/everyth...@googlegroups.com/msg91022.html 

One way to think about local vs. non-local explanations is to imagine running a *simulation* of a Bell type experiment, using three or more separate computers that are each responsible for simulating what's going on in a localized region of space, say the location of experimenter A ('Alice'), the location of experimenter B ('Bob'), and the location of the emitter midway between them. The computer simulating the location of the emitter has to run some algorithm that assigns states to the two emitted particles (the algorithm is allowed to involve something like a random number generator, it need not be deterministic), and then it can transmit some or all of that information to the computers simulating the locations of Alice and Bob. Then once the computer simulating Alice's location receives that information about the state of the simulated particle arriving there, it runs some algorithm to decide what detector setting Alice selects, and what happens in that local region when she measures the particle with that detector setting (again we are allowed to use a random number generator), and the computer simulating Bob's location does the same. If we want to simulate a model of physics that obeys locality, then computers simulating events with a spacelike separation, like Alice performing her measurement and Bob performing his, cannot be in communication at the moment they each compute the local outcome at their own location. And if we want to avoid superdeterminism, the computer simulating the emitter cannot have any way to predict in advance what measurement setting Alice and Bob are going to use at their own locations--over many trials, the states it assigns to the particles on each trial cannot be statistically correlated with the future choices of detector settings by Alice and Bob on that trial.

A simulation based on a MWI style toy model could respect both of these conditions, locality (no communication between computers when they are determining the results of events that are supposed to be at a spacelike separation, like Alice's measurement and Bob's measurement) and non superdeterminism (the computer simulating the emitter generates the states to send to Alice and Bob with no advanced knowledge of what detector setting they are going to choose in the future). The twist with the MWI is that the computers simulating Alice and Bob's locations don't generate unique outcomes, but rather a collection of different outcomes for different simulated copies of Alice and Bob. If all the copies then send signals reporting their results back towards the location of the emitter at the midpoint between them, then the "matching" between copies of Alice and copies of Bob can be done in the computer simulating the location of the emitter, when it has had time to receive signals traveling at the speed of light or slower from the Alice-computer and the Bob-computer, thus there is no violation of the locality condition. If for example there is a third observer, Carla, who is at the location of the emitter and waiting to receive signals from Alice and Bob, Carla only gets split into different copies receiving different possible messages from Alice and Bob at point when Alice's measurement and Bob's measurement are both in her past light cone.

In contrast, no simulated rules for a *single* world could reproduce Bell inequality violating statistics in a situation like this with 3 distinct computers for each local region, not if we imposed the constraints of locality and non-superdeterminism described above. I think you claimed earlier that your fGRW idea is supposed to be local, but if you tried to formalize it sufficiently to build rules for a simulation subject to these constraints, it wouldn't be able reproduce Bell inequality violating statistics either.








 
They are, therefore, otiose, and MWI can be discarded.

Bruce

--
You received this message because you are subscribed to the Google Groups "Everything List" group.
To unsubscribe from this group and stop receiving emails from it, send an email to everything-li...@googlegroups.com.

Stathis Papaioannou

unread,
Dec 22, 2021, 6:21:24 PM12/22/21
to everyth...@googlegroups.com
In MWI, one person saw the atom decay and the other did not. They are not observable to each other, but they observe themselves, obviously.
--
Stathis Papaioannou

John Clark

unread,
Dec 22, 2021, 7:03:06 PM12/22/21
to 'Brent Meeker' via Everything List
On Wed, Dec 22, 2021 at 6:21 PM Stathis Papaioannou <stat...@gmail.com> wrote:


> In MWI, one person saw the atom decay and the other did not. They are not observable to each other, but they observe themselves, obviously.

Obviously. 

John K Clark

 

Bruce Kellett

unread,
Dec 22, 2021, 7:33:59 PM12/22/21
to Everything List
On Thu, Dec 23, 2021 at 10:01 AM Jesse Mazer <laser...@gmail.com> wrote:
On Wed, Dec 22, 2021 at 4:54 PM Bruce Kellett <bhkel...@gmail.com> wrote:
On Wed, Dec 22, 2021 at 10:12 PM smitra <smi...@zonnet.nl> wrote:
On 21-12-2021 22:48, Bruce Kellett wrote:

> Rubbish. If there were a common cause, then that would have to depend
> on the final polarizer orientations. And those are not known at the
> time of creation of the entangled pair. You are, then, back with some
> non-local influence (or retro-causation).

The setting of the polarizers will be the result of some physical
process. Whatever you specify for that process should be included in the
analysis of the problem. But when you do so, it's inevitable that in an
MWI analysis, there is not going to be any nonlocal effect other than
trivial common cause effects.

I see. So in desperation you resort to the superdeterminism escape.

I don't think Saibal was referring to superdeterminism? Or are you suggesting the MWI version of locality involves superdeterminism? If so that's wrong, superdeterminism involves some special constraint on initial conditions such that variables associated with the entangled particles (hidden or non-hidden) at the moment they are sent out in opposite directions are correlated in advance with the future choices of detector settings by the experimenters.

MWI is not necessary for the understanding of the correlations of entangled particles, as my simple example shows. In an actual experiment, the analysis is identical in many-worlds and collapse models. The additional worlds in MWI add nothing to the explanation.

They allow it to be local without superdeterminism, because the "matching" of local worlds can be done at a point in spacetime that has both experimenter's measurements in its past light cone, I gave you a toy model demonstrating how this can work in my post at https://www.mail-archive.com/everyth...@googlegroups.com/msg91022.html 

One way to think about local vs. non-local explanations is to imagine running a *simulation* of a Bell type experiment, using three or more separate computers that are each responsible for simulating what's going on in a localized region of space, say the location of experimenter A ('Alice'), the location of experimenter B ('Bob'), and the location of the emitter midway between them. The computer simulating the location of the emitter has to run some algorithm that assigns states to the two emitted particles

The whole point of the entanglement is that there are no separately assigned states to the two particles. They are in an entangled, non-separable, state. So that the particle that goes to A is non-locally linked to the particle that goes to B.
There is a simple rule here. If the particles interact only locally, then the joint state is separable. If the state is non-separable, the interactions are non-local. All local states are separable. Non-separable states are non-local. Modus tollens.


(the algorithm is allowed to involve something like a random number generator, it need not be deterministic), and then it can transmit some or all of that information to the computers simulating the locations of Alice and Bob. Then once the computer simulating Alice's location receives that information about the state of the simulated particle arriving there, it runs some algorithm to decide what detector setting Alice selects, and what happens in that local region when she measures the particle with that detector setting (again we are allowed to use a random number generator), and the computer simulating Bob's location does the same. If we want to simulate a model of physics that obeys locality, then computers simulating events with a spacelike separation, like Alice performing her measurement and Bob performing his, cannot be in communication at the moment they each compute the local outcome at their own location. And if we want to avoid superdeterminism, the computer simulating the emitter cannot have any way to predict in advance what measurement setting Alice and Bob are going to use at their own locations--over many trials, the states it assigns to the particles on each trial cannot be statistically correlated with the future choices of detector settings by Alice and Bob on that trial.

A simulation based on a MWI style toy model could respect both of these conditions, locality (no communication between computers when they are determining the results of events that are supposed to be at a spacelike separation, like Alice's measurement and Bob's measurement) and non superdeterminism (the computer simulating the emitter generates the states to send to Alice and Bob with no advanced knowledge of what detector setting they are going to choose in the future). The twist with the MWI is that the computers simulating Alice and Bob's locations don't generate unique outcomes, but rather a collection of different outcomes for different simulated copies of Alice and Bob. If all the copies then send signals reporting their results back towards the location of the emitter at the midpoint between them, then the "matching" between copies of Alice and copies of Bob


Again, the trouble here is that your model requires this "matching" operation, which is not present in the actual experiments, such as that of Aspect. In the real-world experiments, there is no further interaction at the point where Alice and Bob meet. All they have to do is calculate the correlations implied by the data in their respective lab books. The Alice and Bob that meet up (and there are 2^N such Alice-Bob pairs in MWI) they simply compare notes. The non-locality of the entangled state means that they were always both in the same world, so no "matching up" is required. There is no work for the third central computer to do in the real-world experiments.


can be done in the computer simulating the location of the emitter, when it has had time to receive signals traveling at the speed of light or slower from the Alice-computer and the Bob-computer, thus there is no violation of the locality condition. If for example there is a third observer, Carla, who is at the location of the emitter and waiting to receive signals from Alice and Bob, Carla only gets split into different copies receiving different possible messages from Alice and Bob at point when Alice's measurement and Bob's measurement are both in her past light cone.

In contrast, no simulated rules for a *single* world could reproduce Bell inequality violating statistics in a situation like this with 3 distinct computers for each local region, not if we imposed the constraints of locality and non-superdeterminism described above. I think you claimed earlier that your fGRW idea is supposed to be local, but if you tried to formalize it sufficiently to build rules for a simulation subject to these constraints, it wouldn't be able reproduce Bell inequality violating statistics either.

The trouble with these computer simulation models is that the conditions are generally set in a way that does not reflect the real-world experiment. Given a simulation, you can set it up to prove anything you want. The requirement is that you provide a local explanation of something like the actual Aspect experiment -- where there is no third computer or further interaction when the data from the separate ends of the experiment are used to calculate correlations (or probabilities).

Incidentally it has never been claimed that fGRW gives a local account of these experiments. fGRW is Lorentz invariant and fully relativistic. But it also respects the non-locality of quantum mechanics. I think that since you have not refuted my no-go theorem for locality preservation in MWI, there is no real point in my finding all the errors made in the endless attempts to give local MWI accounts of Bell inequality violations.

Bruce

smitra

unread,
Dec 23, 2021, 5:55:13 AM12/23/21
to everyth...@googlegroups.com
On 22-12-2021 22:54, Bruce Kellett wrote:
> On Wed, Dec 22, 2021 at 10:12 PM smitra <smi...@zonnet.nl> wrote:
>
>> On 21-12-2021 22:48, Bruce Kellett wrote:
>>>
>>> In general, that is not true. When both Alice and Bob set their
>>> polarizers randomly while the particles are in flight, the fact
>> that
>>> Alice might get |up> tells her nothing about what Bob will get at
>> some
>>> randomly different polarizer orientation. You seem to be stuck
>> with
>>> thinking in terms of parallel polarizer orientations.
>>
>> It's not true only when the polarizers are orthogonal. Whenever the
>> polarizers are not orthogonal, Alice will gain some amount of
>> information about what Bob will find given the result of her
>> measurement. For Bob, the probability of finding up or down are
>> always
>> 1/2, but after Alice makes her measurement, the conditional
>> probability
>> of what Bob will find, given her measurement result will not be
>> equal to
>> 1/2 for both outcomes if her polarizer was not orthogonal to that of
>>
>> Bob, so Alice will have gained information about Bob's measurement
>> result.
>
> The conditional probability you refer to is defined only non-locally.
>

There are no nontrivial nonlocal effects in the MWI. Once you specify
how Alice and Bob decide to choose their polarizers, you can analyze the
flow of information. If you do that within the MWI framework there won't
by any nonlocal effects apart from common cause effects where
information created at one spacetime point ended up travelling in two
directions via local processes and ended up creating correlations in
spacelike separated systems.
As Jesse Mazer pints out this has nothing to do with superdeterminism.
You can e.g. let Bon And Alice do additional spin measurements on other
(non-entangled) electrons and use the random results of those to
determine the orientation of their polarizers. Thing is that you need to
choose some physical process for this. There is then no appeal to the
setting of the polarizer having been pre-determined in a way to explain
the correlations, so this is not an appeal to superdeterminism.

Collapse models invoke new, as of yet unobserved physics, at scales
where our present theories of physics are very solid. While such
collapse theories could be correct, they are not motivated by an attempt
to solve a problem, like e.g. tensions with experimental results. The
MWI, in contrast, is motivated with problems of standard QM, namely the
unphysical collapse of the wavefunction.

Arguing for collapse models today is like what would have happened if
not Einstein but Maxwell had invited the theory of special relativity.
Some physicists might then have pushed back against that by inventing
the ether to restore the old familiar notion of absolute time.

Also, collapse models may not even get rid of the parallel Worlds. If
the universe is infinite or is infinite in the temporal direction, then
identical copies of us will exist in an infinite number of different but
similar environments. Collapse will then happen, leading to a definite
outcomes in each sector, but you'll be split among all the different
outcomes in different sectors. The only difference with the MWI is then
that according to the MWI the split must happen, while according to
collapse interpretations, a split may happen depending on the large
scale structure of the universe.

Saibal





smitra

unread,
Dec 23, 2021, 6:00:41 AM12/23/21
to everyth...@googlegroups.com
The relevant prediction of the MWI is simply that systems evolve
according to unitary time evolution. No information gets created out of
thin air. If you have collapse, then the random element of the collapse
amounts to information added to the system.

Saibal



Brent Meeker

unread,
Dec 23, 2021, 2:01:57 PM12/23/21
to everyth...@googlegroups.com
As I see it, the only problem MWI solves is to maintain determinism
contrary to all experience by saying, everything happens but not where
anyone can see it.  Which is as if Einstein had discovered QM instead of
Heisenberg.

Brent
>
>

Brent Meeker

unread,
Dec 23, 2021, 2:03:41 PM12/23/21
to everyth...@googlegroups.com
So does your self-locating uncertainty becoming certain.

Brent

John Clark

unread,
Dec 23, 2021, 2:12:39 PM12/23/21
to 'Brent Meeker' via Everything List
On Thu, Dec 23, 2021 at 2:01 PM Brent Meeker <meeke...@gmail.com> wrote:

> As I see it, the only problem MWI solves is to maintain determinism

And solves the measurement problem. And gets rid of the ridiculous Heisenberg cut.

John K Clark    See what's on my new list at  Extropolis
whc





Bruce Kellett

unread,
Dec 23, 2021, 5:01:28 PM12/23/21
to Everything List
On Thu, Dec 23, 2021 at 9:55 PM smitra <smi...@zonnet.nl> wrote:
On 22-12-2021 22:54, Bruce Kellett wrote:
> On Wed, Dec 22, 2021 at 10:12 PM smitra <smi...@zonnet.nl> wrote:
>
>> On 21-12-2021 22:48, Bruce Kellett wrote:
>>>
>>> In general, that is not true. When both Alice and Bob set their
>>> polarizers randomly while the particles are in flight, the fact that
>>> Alice might get |up> tells her nothing about what Bob will get at some
>>> randomly different polarizer orientation. You seem to be stuck with
>>> thinking in terms of parallel polarizer orientations.
>>
>> It's not true only when the polarizers are orthogonal. Whenever the
>> polarizers are not orthogonal, Alice will gain some amount of
>> information about what Bob will find given the result of her
>> measurement. For Bob, the probability of finding up or down are always
>> 1/2, but after Alice makes her measurement, the conditional probability
>> of what Bob will find, given her measurement result will not be equal to
>> 1/2 for both outcomes if her polarizer was not orthogonal to that of 
>> Bob, so Alice will have gained information about Bob's measurement result.
>
> The conditional probability you refer to is defined only non-locally.
>

There are no nontrivial nonlocal effects in the MWI.

That is what remains to be proved.

Once you specify
how Alice and Bob decide to choose their polarizers, you can analyze the
flow of information

As I understand it, Deutsch and Hayden attempted this in arXiv:quant-ph/9906007. This idea has proved to be unsuccessful. One of the problems being that their construction bears little relationship to what happens in the laboratory. One can make up toy models to demonstrate almost anything. However, in order to be useful, such models must be closely tied to laboratory experience.


If you do that within the MWI framework there won't
by any nonlocal effects apart from common cause effects where
information created at one spacetime point ended up travelling in two
directions via local processes and ended up creating correlations in
spacelike separated systems.

>>>> In the MWI
>>>> there is no such mysterious gain of information due to the correlation
>>>> being caused by common cause when the entangled pair is created
>>>
>>> Rubbish. If there were a common cause, then that would have to depend
>>> on the final polarizer orientations. And those are not known at the
>>> time of creation of the entangled pair. You are, then, back with some
>>> non-local influence (or retro-causation).
>>
>> The setting of the polarizers will be the result of some physical
>> process. Whatever you specify for that process should be included in the
>> analysis of the problem. But when you do so, it's inevitable that in an
>> MWI analysis, there is not going to be any nonlocal effect other than
>> trivial common cause effects.

As I have said. That is your contention, but it has yet to be proved. The method of setting the polarizers can be included if you must, but this is ultimately irrelevant to the main issues here. It seems that you introduce it only as a distraction.


> I see. So in desperation you resort to the superdeterminism escape.
> MWI is not necessary for the understanding of the correlations of
> entangled particles, as my simple example shows. In an actual
> experiment, the analysis is identical in many-worlds and collapse
> models. The additional worlds in MWI add nothing to the explanation.
> They are, therefore, otiose, and MWI can be discarded.


As Jesse Mazer pints out this has nothing to do with superdeterminism.
You can e.g. let Bon And Alice do additional spin measurements on other
(non-entangled) electrons and use the random results of those to
determine the orientation of their polarizers. Thing is that you need to
choose some physical process for this. There is then no appeal to the
setting of the polarizer having been pre-determined in a way to explain
the correlations, so this is not an appeal to superdeterminism.

I am not arguing for superdeterminism. It just seemed that insistence on including the method for setting polarizer angles was akin to the arguments made be advocates of superdeterminism.


Collapse models invoke new, as of yet unobserved physics, at scales
where our present theories of physics are very solid. While such
collapse theories could be correct, they are not motivated by an attempt
to solve a problem, like e.g. tensions with experimental results. The
MWI, in contrast, is motivated with problems of standard QM, namely the
unphysical collapse of the wavefunction.

This discussion has nothing to do with one's philosophical preference for non-collapse over collapse models. The issue is whether or not MWI can give a local account of the Bell correlations. A simple analysis based on actual laboratory experience shows that many-worlds cannot accomplish this. Concocting artificial models is a well established path for proving anything at all. Your models must be based in laboratory experience. Jesse's computer models are totally unrealistic, and bear no relation to real experiments.


Arguing for collapse models today is like what would have happened if
not Einstein but Maxwell had invited the theory of special relativity.
Some physicists might then have pushed back against that by inventing
the ether to restore the old familiar notion of absolute time.

Also, collapse models may not even get rid of the parallel Worlds. If
the universe is infinite or is infinite in the temporal direction, then
identical copies of us will exist in an infinite number of different but
similar environments. Collapse will then happen, leading to a definite
outcomes in each sector, but you'll be split among all the different
outcomes in different sectors. The only difference with the MWI is then
that according to the MWI the split must happen, while according to
collapse interpretations, a split may happen depending on the large
scale structure of the universe.

That really is a confusion of ideas, all of which are completely speculative. I think it behooves us to keep a little more to the point.

Bruce

Bruce Kellett

unread,
Dec 23, 2021, 5:06:33 PM12/23/21
to Everything List
Of course, information is added in MWI as well. Every time you get a definite result from an experiment you learn something new. Physics is based on the gathering of new information from experiments. Obtaining a definite result is characteristic of both many-worlds and collapse models. But again, this is irrelevant to the optic under discussion.

Bruce

Dirk Van Niekerk

unread,
Dec 24, 2021, 3:53:27 PM12/24/21
to Everything List


On Tuesday, December 21, 2021 at 9:01:30 AM UTC-8 johnk...@gmail.com wrote:
On Mon, Dec 20, 2021 at 10:58 PM Stathis Papaioannou <stat...@gmail.com> wrote:

>but there are events such as the decay of an atom within a half life period that one version of you will see and another version of you will see, which is interpreted as a 1/2 probability of you seeing the atom decay, if you have a normal human brain without telepathic communication with other copies.
 
All versions of "you" that live in worlds that have the same fundamental laws of physics (and those that don't would be so different they probably wouldn't deserve to be called "you") would agree that Neptunium 240 has a half-life of one hour, in other words that mode of decay would be the most common and most of "you" in the multi-verse would see an atom of Neptunium decay at around the one hour mark. But most does not mean all and if we're talking about one particular Neptunium 240 atom a minority of "you" will not see it decay after 5 hours even though you know it's half life is only one hour, and a very tiny minority will not see it decay even after 5 million years, and another very tiny minority of "you" will see it decay after only 5 nanoseconds.

You may ask, how different can "you" be before it no longer deserves the right to be called "you"? I admit that limit is somewhat arbitrary, but the important point is that whatever limit you choose, as long as it's consistent, it makes no difference if high precision is demanded for something to be called "you" or if extreame sloppiness can be tolerated, either way it will still remain true that there will be more "yous" near the center of the Bell Curve than at the trailing edges.
John K Clark    See what's on my new list at  Extropolis
Imagine one physicist starts a series of quantum experiments.  Each experiment has two outcomes with predicted probabilities of A=p1 and B=p2.  In MWI, after doing an arbitrarily large number of these experiments the end observer in each branch now tabulates their observations and find that they have measured outcome A p1 times and outcome B p2 times.  Each observer therefore must have followed a branching path that lead to this outcome.  What in the MWI and the Schroedinger equation determined that each observer would find those probabilities (other than arbitrarily invoking the Born rule).  What in MWI prevents that a large number of observers will report a probability for either A or B as 100%.

Dirk 

emc

Bruce Kellett

unread,
Dec 24, 2021, 5:07:43 PM12/24/21
to Everything List
On Sat, Dec 25, 2021 at 7:53 AM Dirk Van Niekerk <leeu...@gmail.com> wrote:

Imagine one physicist starts a series of quantum experiments.  Each experiment has two outcomes with predicted probabilities of A=p1 and B=p2.  In MWI, after doing an arbitrarily large number of these experiments the end observer in each branch now tabulates their observations and find that they have measured outcome A p1 times and outcome B p2 times.  Each observer therefore must have followed a branching path that lead to this outcome.  What in the MWI and the Schroedinger equation determined that each observer would find those probabilities (other than arbitrarily invoking the Born rule).  What in MWI prevents that a large number of observers will report a probability for either A or B as 100%.

The characteristic of MWI is that every outcome occurs in its own branch for every trial. If there are just two outcomes, A and B in your case, then after N trials there will be 2^N copies of the observer -- each with an individual sequence of A B results, covering all possible 2^N sequences for N trials. According to the binomial theorem, for large N the relative proportions of A and B in these sequences will peak around 50/50. In other words, the majority of the copies at the end of this experiment will find data suggesting a probability of 0.5 for A, and 0.5 for B. There will, of course, be a number of outliers with discrepant statistics, such as sequences dominated by As or by Bs. But these form a vanishing proportion in the limit of large N.

The obvious trouble with this is that the majority find a 50/50 ratio, regardless of the actual specified probabilities of A=p1 and B=p2. There is, in fact, no way in which unmodified MWI can get data that reflects the actual probabilities when these differ significantly from p1=p2=0.5. This is one of main main objections to MWI -- direct confrontation with the data clearly falsifies the theory.

Of course, people have come up with various fixes to MWI to overcome this. One popular way is to simply add additional branches on each trial so that the proportions reflect the required probabilities. Zurek and Carroll have varieties of this approach. The trouble here is that this is completely ad hoc. and it also turns out to be circular, because the only way in which one can know how many additional branches to add to each outcome is to look to the Born probabilities -- probabilities that are not available in the raw Schrodinger equation.

Other possible fixes have been tried, but none can actually overcome the basic problem that MWI is inconsistent with real-world data.

Bruce

John Clark

unread,
Dec 24, 2021, 5:08:18 PM12/24/21
to 'Brent Meeker' via Everything List
On Fri, Dec 24, 2021 at 3:53 PM Dirk Van Niekerk <leeu...@gmail.com> wrote:

>> All versions of "you" that live in worlds that have the same fundamental laws of physics (and those that don't would be so different they probably wouldn't deserve to be called "you") would agree that Neptunium 240 has a half-life of one hour, in other words that mode of decay would be the most common and most of "you" in the multi-verse would see an atom of Neptunium decay at around the one hour mark. But most does not mean all and if we're talking about one particular Neptunium 240 atom a minority of "you" will not see it decay after 5 hours even though you know it's half life is only one hour, and a very tiny minority will not see it decay even after 5 million years, and another very tiny minority of "you" will see it decay after only 5 nanoseconds. You may ask, how different can "you" be before it no longer deserves the right to be called "you"? I admit that limit is somewhat arbitrary, but the important point is that whatever limit you choose, as long as it's consistent, it makes no difference if high precision is demanded for something to be called "you" or if extreame sloppiness can be tolerated, either way it will still remain true that there will be more "yous" near the center of the Bell Curve than at the trailing edges.

> Imagine one physicist starts a series of quantum experiments.  Each experiment has two outcomes with predicted probabilities of A=p1 and B=p2.  In MWI, after doing an arbitrarily large number of these experiments the end observer in each branch now tabulates their observations and find that they have measured outcome A p1 times and outcome B p2 times.  Each observer therefore must have followed a branching path that lead to this outcome.  What in the MWI and the Schroedinger equation determined that each observer would find those probabilities (other than arbitrarily invoking the Born rule). 

First of all nobody arbitrarily invokes the Born Rule, thanks to experiment we know for a fact that it works, we also know it is the only way to get probabilities out of the Schrodinger equation in which all the probabilities are positive and they all add up to 1, after all that part is not even controversial because the Born Rule is really just Pythagoras's Theorem on steroids. And classical physics is a very good approximation of the Schrodinger equation when things get large, so If you want to calculate the odds of a coin flip you can forget quantum mechanics entirely and just use classical physics, it says that if you flip a fair coin there's no reason to expect one side of the coin to be favored over another, so the odds must be 50-50.
 
> What in MWI prevents that a large number of observers will report a probability for either A or B as 100%.
 
Suppose there was a being with a godlike intellect and knew the quantum wave function of the entire multiverse, since the Schrodinger equation is entirely deterministic couldn't he just forget about probabilities entirely and predict with 100% confidence which branch of the multiverse he will be on and thus predict with 100% certainty if he will see an atom of Neptunium 240 decay in the next hour or not? No he could not, and the reason he couldn't is because he would know everything there is to know about the multiverse except for which branch he is on. Even with infinite intelligence he couldn't say "I will now answer the question  "which ONE and only ONE branch will I end up being on after I change from 1 to 2?".  And the reason he couldn't do that is because even infinite intelligence is not good enough to  provide an answer to gibberish, and despite the existence of a question mark at the end "which ONE and only ONE branch  will I end up being on after I change from 1 to 2?"  is not a question, it is self contradictory gibberish.  

John Clark

unread,
Dec 24, 2021, 5:24:51 PM12/24/21
to 'Brent Meeker' via Everything List
On Fri, Dec 24, 2021 at 5:07 PM Bruce Kellett <bhkel...@gmail.com> wrote:

> The obvious trouble with this is that the majority find a 50/50 ratio, regardless of the actual specified probabilities of A=p1 and B=p2.

I don't know how you figure that! Some human beings are over 8 feet tall but not many, if you picked a population of 100 people at random you might eventually find a population in which 50% were over 8 feet tall, but I think the sun would've turned into a red giant before you found it. And by the way, the Many Worlds advocates calculate probabilities exactly the same way Many Worlds opponents do, if they have been doing it wrong all these years then so has everybody else.

John K Clark    See what's on my new list at  Extropolis
ddu






 

Bruce Kellett

unread,
Dec 24, 2021, 5:40:26 PM12/24/21
to Everything List
On Sat, Dec 25, 2021 at 9:24 AM John Clark <johnk...@gmail.com> wrote:
On Fri, Dec 24, 2021 at 5:07 PM Bruce Kellett <bhkel...@gmail.com> wrote:

> The obvious trouble with this is that the majority find a 50/50 ratio, regardless of the actual specified probabilities of A=p1 and B=p2.

I don't know how you figure that!

I suggest you look up a standard reference on the binomial distribution. The 50/50 ratio is a consequence of the fact that there are just two possibilities on each trial.

 
Some human beings are over 8 feet tall but not many, if you picked a population of 100 people at random you might eventually find a population in which 50% were over 8 feet tall, but I think the sun would've turned into a red giant before you found it. And by the way, the Many Worlds advocates calculate probabilities exactly the same way Many Worlds opponents do, if they have been doing it wrong all these years then so has everybody else.

You can estimate probabilities from relative proportions, but that is not a good definition of probability.

Bruce

John Clark

unread,
Dec 24, 2021, 6:15:31 PM12/24/21
to 'Brent Meeker' via Everything List
On Fri, Dec 24, 2021 at 5:40 PM Bruce Kellett <bhkel...@gmail.com> wrote:

> I suggest you look up a standard reference on the binomial distribution. The 50/50 ratio is a consequence of the fact that there are just two possibilities on each trial.

You're completely ignoring the amplitude of the quantum wave function, in other words you're completely ignoring Schrodinger's equation. And no Many World's advocate, and no physicist alive for that matter, believes you can figure out probabilities from branch counting. You are allowed to assign equal probabilities to branches ONLY when they have identical amplitudes; that's why we must use amplitude squared when we want to figure out probabilities.

John K Clark    See what's on my new list at  Extropolis
bcx



Bruce Kellett

unread,
Dec 24, 2021, 8:20:07 PM12/24/21
to Everything List
On Sat, Dec 25, 2021 at 10:15 AM John Clark <johnk...@gmail.com> wrote:
On Fri, Dec 24, 2021 at 5:40 PM Bruce Kellett <bhkel...@gmail.com> wrote:

> I suggest you look up a standard reference on the binomial distribution. The 50/50 ratio is a consequence of the fact that there are just two possibilities on each trial.

You're completely ignoring the amplitude of the quantum wave function, in other words you're completely ignoring Schrodinger's equation.

If you check carefully, you will find that Schrodinger's equation is insensitive to the amplitude of the wave function. The wave function splits according to the eigenfunctions for the allowed values (The number of components of the wave function is given by the dimension of the relevant Hilbert space.). But the wave function splits only according to the number of eigenvalues, ignoring the actual coefficients of these components of the wave function.


And no Many World's advocate, and no physicist alive for that matter, believes you can figure out probabilities from branch counting. You are allowed to assign equal probabilities to branches ONLY when they have identical amplitudes; that's why we must use amplitude squared when we want to figure out probabilities.

That is one of the reasons that the Born rule is not derivable from the Schrodinger equation or the wave function. If you look at Carroll and Sebens, they acknowledge that their prescription boils down to simple branch counting -- after the number of branches is supplemented to be in the proportion required by the Born rule, with each branch having the same amplitude. If you use self-locating uncertainty as your measure of probability, you have to resort to branch counting in the final analysis. Just think about it for a while........

Bruce

John Clark

unread,
Dec 24, 2021, 10:18:31 PM12/24/21
to 'Brent Meeker' via Everything List
On Fri, Dec 24, 2021 at 8:20 PM Bruce Kellett <bhkel...@gmail.com> wrote:

>> You're completely ignoring the amplitude of the quantum wave function, in other words you're completely ignoring Schrodinger's equation.

> If you check carefully, you will find that Schrodinger's equation is insensitive to the amplitude of the wave function.

What in the world are you talking about. Determining the quantum wave function is the only reason the Schrodinger equation is of any use, and the Born Rule is the only reason the quantum wave function is of any use.   

> That is one of the reasons that the Born rule is not derivable from the Schrodinger equation or the wave function.

As I keep saying, the Born rule does not need to be derived from anything nor does it need to be assumed  because we already know from a huge number of experiments that it is correct; if it wasn't the computer I'm typing this on wouldn't work, the modern world economy wouldn't work either.  
 
If you look at Carroll and Sebens, they acknowledge that their prescription boils down to simple branch counting

Nope. From page 143 of Sean Carroll's book "Something Deeply Hidden"

"It's easy to show that this idea known as branch counting can't possibly work"

> after the number of branches is supplemented to be in the proportion required by the Born rule

After the number of branches is amplified or reduced according to the amplitude of the wave function.

 John K Clark    See what's on my new list at  Extropolis
bcq




 

Bruce Kellett

unread,
Dec 24, 2021, 11:57:30 PM12/24/21
to Everything List
On Sat, Dec 25, 2021 at 2:18 PM John Clark <johnk...@gmail.com> wrote:
On Fri, Dec 24, 2021 at 8:20 PM Bruce Kellett <bhkel...@gmail.com> wrote:

>> You're completely ignoring the amplitude of the quantum wave function, in other words you're completely ignoring Schrodinger's equation.

> If you check carefully, you will find that Schrodinger's equation is insensitive to the amplitude of the wave function.

What in the world are you talking about. Determining the quantum wave function is the only reason the Schrodinger equation is of any use, and the Born Rule is the only reason the quantum wave function is of any use. 

The wave function is a vector in Hilbert space. The Schrodinger equation determines the time evolution of the vector. An observable quantity is represented by a Hermitian operator in this Hilbert space. The space is spanned by a set of basis vectors that are conveniently taken to be the eigenvectors of the related measurement operator. Any wave function can be expanded in terms of this set of basis vectors. There are as many of them as there are distinct possible outcomes from a measurement of the corresponding operator (the dimension of the Hilbert space).

In MWI, it is assumed that in any measurement all possible outcomes are realized, albeit in different worlds. Any vector in a Hilbert space is expanded in terms of the same set of eigenvectors, so has the same set of possible outcomes. This set is independent of the coefficients determining different vectors in the base space. In other words, the set of possible results for any measurement involving a particular operator and base space is independent of the amplitude of any particular basis vector in the wave function.

In the spin measurement case, there are two possible outcomes, |up> or |down>, so the Hilbert space is two dimensional. Any vector in this space can be expanded in terms of these basis vectors:

        |psi> = a|up>  + b|down>

and the possible results are up or down, independent of the coefficients (or amplitudes) a and b.


> That is one of the reasons that the Born rule is not derivable from the Schrodinger equation or the wave function.

As I keep saying, the Born rule does not need to be derived from anything nor does it need to be assumed  because we already know from a huge number of experiments that it is correct; if it wasn't the computer I'm typing this on wouldn't work, the modern world economy wouldn't work either.

The Born rule works. But that does not mean that it does not need to be derived or postulated.

 
If you look at Carroll and Sebens, they acknowledge that their prescription boils down to simple branch counting

Nope. From page 143 of Sean Carroll's book "Something Deeply Hidden"

"It's easy to show that this idea known as branch counting can't possibly work"

That refers to naive branch counting. When the number of branches is increased so that all the amplitudes are equal -- equal probability for each branch -- then the probability of a particular result is proportional to the number of branches giving this result. This is effectively branch counting, and Carrol admits as much in one of his later papers with Sebens.


> after the number of branches is supplemented to be in the proportion required by the Born rule

After the number of branches is amplified or reduced according to the amplitude of the wave function.

That is essentially what I said. There must be a form of branch counting if self-locating uncertainty is going to work to give you the probability.

Bruce

spudb...@aol.com

unread,
Dec 25, 2021, 1:05:01 AM12/25/21
to bhkel...@gmail.com, everyth...@googlegroups.com
The amplitude of the wave function may be far broader then the worthies on this mailing-list have so far proposed. That what is quantum, specifically a process space where the Multi basically like Hugh Everett + John Wheeler evoked is like is something like a spaghetti chart if we postulate observables?

Bear Witness on this Hallowed Day! A couple of neighbor's from master Clark's Texas A&M produced this ARXIV publication that tries to lighten these issues of Born + Heisenberg + Schrodinger  and I am bit non-plus'd to not see Wigner's' Friend astride the head of Mr. S's Cat? 

"Introducing the ‘process dimension’ One way to develop a more thorough understanding of a situation is to build a model. Models can be physical (like an architect’s scale model of a building), or they can be visual or conceptual, like a diagram (which requires more imagination to appreciate). Model-building always involves making some simplifications, in order to reduce unnecessary complications and bring out essential details of the situation being modeled. Now we’re trying to model a situation where events that occur outside of space and time “collide” with our universe. But it’s difficult enough to visualize four-dimensional spacetime, let alone events that occur somewhere outside! To get a hold of this idea to begin with, we should try to simplify as much as possible, without excluding the essential details. In doing this, we may once again cite the example of Einstein, who is often quoted as saying “Everything should be made as simple as possible, but no simpler.” 1"

To wit, Make The Jump to Process Space...



--
You received this message because you are subscribed to the Google Groups "Everything List" group.
To unsubscribe from this group and stop receiving emails from it, send an email to everything-li...@googlegroups.com.
To view this discussion on the web visit

John Clark

unread,
Dec 25, 2021, 10:25:24 AM12/25/21
to 'Brent Meeker' via Everything List
Bruce Kellett <bhkel...@gmail.com> wrote:

>> What in the world are you talking about. Determining the quantum wave function is the only reason the Schrodinger equation is of any use, and the Born Rule is the only reason the quantum wave function is of any use. 

> The wave function is a vector in Hilbert space. The Schrodinger equation determines the time evolution of the vector.

Right. And that's why the Born Rule is just Pythagoras's theorem in action.
 
> >> after the number of branches is supplemented to be in the proportion required by the Born rule
 
>> After the number of branches is amplified or reduced according to the amplitude of the wave function.

> That is essentially what I said. There must be a form of branch counting if self-locating uncertainty is going to work to give you the probability.

If one branch does not correspond to just one integer but instead some branches must be "supplemented" more than others then you're not doing branch counting, in fact you're not doing "counting" of any sort. Actually it's even worse than that because when you're counting physical things and not just numbers you have to be sure that the units are the same, that's why 2 apples plus 2 more apples does NOT equal 4 oranges. A branch is not a probability, to an observer on a rare low amplitude branch things would seem just as real to him as an observer in a common high amplitude branch. And because a branch is not a probability you can't add up branches and expect to get a probability.  

> In MWI, it is assumed that in any measurement all possible outcomes are realized, albeit in different worlds.

Yes.

> Any vector in a Hilbert space is expanded in terms of the same set of eigenvectors, so has the same set of possible outcomes.

I'm not talking about Hilbert space and I'm not talking about eigenvectors, I'm talking about probabilities. And to determine probability if you must use a variable conversion factor that is proportional to the square of the magnitude of the quantum wave function that not only modifies the weight each branch has in determining the total probability but also changes the basic nature of the thing that you're counting, then the claim it is branch "counting" is like saying if I had some cream I could have strawberries and cream, if I had some strawberries. 

John K Clark    See what's on my new list at  Extropolis
bcn



 

Brent Meeker

unread,
Dec 25, 2021, 7:43:57 PM12/25/21
to everyth...@googlegroups.com
The ad hockery can be overcome by assuming there are an enormous number of macroscopically indistinguishable branches even before the experiment...which would be consistent, since their are always an enormous number of microscopic events being amplified enough to leave a record but not enough to affect our perceptions.  So an experiment for which the Born rule predicts a 49/51 split can split the large number of existing branches in this proportion.  Julian Barbour uses a metaphor like this as the state of the world being like river of equivalence classes, we regard as a single stream until it's split into different channels.  What is missing in this picture is some mechanism for the splittiing.  The Schroedinger equation that predicts the 49/51 split doesn't say anything about interacting with the different threads of this stream.  So the advocates claim that it's "just the Schroedinger equation" doesn't seem adequate.

Brent


Other possible fixes have been tried, but none can actually overcome the basic problem that MWI is inconsistent with real-world data.

Bruce
--
You received this message because you are subscribed to the Google Groups "Everything List" group.
To unsubscribe from this group and stop receiving emails from it, send an email to everything-li...@googlegroups.com.

Bruce Kellett

unread,
Dec 25, 2021, 8:00:10 PM12/25/21
to Everything List
On Sun, Dec 26, 2021 at 11:43 AM Brent Meeker <meeke...@gmail.com> wrote:
On 12/24/2021 2:07 PM, Bruce Kellett wrote:
On Sat, Dec 25, 2021 at 7:53 AM Dirk Van Niekerk <leeu...@gmail.com> wrote:

Imagine one physicist starts a series of quantum experiments.  Each experiment has two outcomes with predicted probabilities of A=p1 and B=p2.  In MWI, after doing an arbitrarily large number of these experiments the end observer in each branch now tabulates their observations and find that they have measured outcome A p1 times and outcome B p2 times.  Each observer therefore must have followed a branching path that lead to this outcome.  What in the MWI and the Schroedinger equation determined that each observer would find those probabilities (other than arbitrarily invoking the Born rule).  What in MWI prevents that a large number of observers will report a probability for either A or B as 100%.

The characteristic of MWI is that every outcome occurs in its own branch for every trial. If there are just two outcomes, A and B in your case, then after N trials there will be 2^N copies of the observer -- each with an individual sequence of A B results, covering all possible 2^N sequences for N trials. According to the binomial theorem, for large N the relative proportions of A and B in these sequences will peak around 50/50. In other words, the majority of the copies at the end of this experiment will find data suggesting a probability of 0.5 for A, and 0.5 for B. There will, of course, be a number of outliers with discrepant statistics, such as sequences dominated by As or by Bs. But these form a vanishing proportion in the limit of large N.

The obvious trouble with this is that the majority find a 50/50 ratio, regardless of the actual specified probabilities of A=p1 and B=p2. There is, in fact, no way in which unmodified MWI can get data that reflects the actual probabilities when these differ significantly from p1=p2=0.5. This is one of main main objections to MWI -- direct confrontation with the data clearly falsifies the theory.

Of course, people have come up with various fixes to MWI to overcome this. One popular way is to simply add additional branches on each trial so that the proportions reflect the required probabilities. Zurek and Carroll have varieties of this approach. The trouble here is that this is completely ad hoc. and it also turns out to be circular, because the only way in which one can know how many additional branches to add to each outcome is to look to the Born probabilities -- probabilities that are not available in the raw Schrodinger equation.

The ad hockery can be overcome by assuming there are an enormous number of macroscopically indistinguishable branches even before the experiment...which would be consistent, since their are always an enormous number of microscopic events being amplified enough to leave a record but not enough to affect our perceptions.  So an experiment for which the Born rule predicts a 49/51 split can split the large number of existing branches in this proportion.  Julian Barbour uses a metaphor like this as the state of the world being like river of equivalence classes, we regard as a single stream until it's split into different channels.  What is missing in this picture is some mechanism for the splittiing.  The Schroedinger equation that predicts the 49/51 split doesn't say anything about interacting with the different threads of this stream.  So the advocates claim that it's "just the Schroedinger equation" doesn't seem adequate.


Generally these approaches are just glorified fairy stories. There is no mechanism that will do any of these wonderful things. If you are allowed to make up stories, you can get away with anything -- such as that MWI gives a local account of Bell correlations.

Bruce

Russell Standish

unread,
Dec 25, 2021, 10:29:24 PM12/25/21
to everyth...@googlegroups.com
The probability of an observer seeing state B given they're in state A
must depend on both A and B. So what you say about a split into B and
¬B giving rise to probabilities of 1/2 cannot be the case in general,
as there would be no dependence on either A or B.

If the measure function (normalisable to probability) is a bilinear
function (which you almost get from the axioms of probability), then
the state space must be a hilbert space, and the probability of A->B
is given by the Born rule. But for the MWI, you already
start with a Hibert space, so even this linearity issue isn't a difficulty.


--

----------------------------------------------------------------------------
Dr Russell Standish Phone 0425 253119 (mobile)
Principal, High Performance Coders hpc...@hpcoders.com.au
http://www.hpcoders.com.au
----------------------------------------------------------------------------

Bruce Kellett

unread,
Dec 25, 2021, 10:58:04 PM12/25/21
to Everything List
Maybe that is the point when all possibilities are realized on every trial.

If the measure function (normalisable to probability) is a bilinear
function (which you almost get from the axioms of probability), then
the state space must be a hilbert space, and the probability of A->B
is given by the Born rule. But for the MWI, you already
start with a Hibert space, so even this linearity issue isn't a difficulty.


I don't understand what you are talking about. If a trial has two possible outcomes, and every outcome is realized in every trial, then after N trials there are 2^N possible sequences of outcomes. These cover all possible binary strings of length N, independent of the probabilities for individual outcomes on any single trial. The binomial theorem (or the law of large numbers) then implies that as N becomes large, in the large majority of sequences you will have approximately equal numbers of each result. If these sequences are used to estimate the probabilities, then most sequences will give p = 0.5 for each result. This is a well-known result.

Bruce

Brent Meeker

unread,
Dec 25, 2021, 11:04:50 PM12/25/21
to everyth...@googlegroups.com


On 12/25/2021 7:24 AM, John Clark wrote:
Bruce Kellett <bhkel...@gmail.com> wrote:

>> What in the world are you talking about. Determining the quantum wave function is the only reason the Schrodinger equation is of any use, and the Born Rule is the only reason the quantum wave function is of any use. 

> The wave function is a vector in Hilbert space. The Schrodinger equation determines the time evolution of the vector.

Right. And that's why the Born Rule is just Pythagoras's theorem in action.
 
> >> after the number of branches is supplemented to be in the proportion required by the Born rule
 
>> After the number of branches is amplified or reduced according to the amplitude of the wave function.

> That is essentially what I said. There must be a form of branch counting if self-locating uncertainty is going to work to give you the probability.

If one branch does not correspond to just one integer but instead some branches must be "supplemented" more than others then you're not doing branch counting, in fact you're not doing "counting" of any sort.

To equate probability with self-location there must be a proportionate number of locations.  Otherwise you would have to suppose there is some "weight" of being in a certain branch.  Neither of these exist in the bare Schroedinger equation.  I'm not saying the MWI is wrong because it needs these supplementary hypotheses; but I am saying its superiority to simply saying one branch happens is less than obvious.

Brent

Actually it's even worse than that because when you're counting physical things and not just numbers you have to be sure that the units are the same, that's why 2 apples plus 2 more apples does NOT equal 4 oranges. A branch is not a probability, to an observer on a rare low amplitude branch things would seem just as real to him as an observer in a common high amplitude branch. And because a branch is not a probability you can't add up branches and expect to get a probability.  

> In MWI, it is assumed that in any measurement all possible outcomes are realized, albeit in different worlds.

Yes.

> Any vector in a Hilbert space is expanded in terms of the same set of eigenvectors, so has the same set of possible outcomes.

I'm not talking about Hilbert space and I'm not talking about eigenvectors, I'm talking about probabilities. And to determine probability if you must use a variable conversion factor that is proportional to the square of the magnitude of the quantum wave function that not only modifies the weight each branch has in determining the total probability but also changes the basic nature of the thing that you're counting, then the claim it is branch "counting" is like saying if I had some cream I could have strawberries and cream, if I had some strawberries. 

John K Clark    See what's on my new list at  Extropolis
bcn



 

--
You received this message because you are subscribed to the Google Groups "Everything List" group.
To unsubscribe from this group and stop receiving emails from it, send an email to everything-li...@googlegroups.com.

Russell Standish

unread,
Dec 26, 2021, 2:38:45 AM12/26/21
to Everything List
On Sun, Dec 26, 2021 at 02:57:51PM +1100, Bruce Kellett wrote:
>
>
> If the measure function (normalisable to probability) is a bilinear
> function (which you almost get from the axioms of probability), then
> the state space must be a hilbert space, and the probability of A->B
> is given by the Born rule. But for the MWI, you already
> start with a Hibert space, so even this linearity issue isn't a difficulty.
>
>
>
> I don't understand what you are talking about. If a trial has two possible
> outcomes, and every outcome is realized in every trial, then after N trials
> there are 2^N possible sequences of outcomes. These cover all possible binary
> strings of length N, independent of the probabilities for individual outcomes
> on any single trial. The binomial theorem (or the law of large numbers) then
> implies that as N becomes large, in the large majority of sequences you will
> have approximately equal numbers of each result. If these sequences are used to
> estimate the probabilities, then most sequences will give p = 0.5 for each
> result. This is a well-known result.

Consider a fair dice, and the two outcomes: a six, and the numbers
1-5. According to your argument, the probability of each outcome is
1/2. Clearly something has gone wrong.

What I'm saying is that the probability must depend on both the
anterior and posterior states. In this thought experiment, the
anterior state is one of maximum ignorance, but the posterior states
have uneven weights.

Bruce Kellett

unread,
Dec 26, 2021, 4:00:24 AM12/26/21
to Everything List
On Sun, Dec 26, 2021 at 6:38 PM Russell Standish <li...@hpcoders.com.au> wrote:
On Sun, Dec 26, 2021 at 02:57:51PM +1100, Bruce Kellett wrote:
>
>     If the measure function (normalisable to probability) is a bilinear
>     function (which you almost get from the axioms of probability), then
>     the state space must be a hilbert space, and the probability of A->B
>     is given by the Born rule. But for the MWI, you already
>     start with a Hibert space, so even this linearity issue isn't a difficulty.
>
>
>
> I don't understand what you are talking about. If a trial has two possible
> outcomes, and every outcome is realized in every trial, then after N trials
> there are 2^N possible sequences of outcomes. These cover all possible binary
> strings of length N, independent of the probabilities for individual outcomes
> on any single trial. The binomial theorem (or the law of large numbers) then
> implies that as N becomes large, in the large majority of sequences you will
> have approximately equal numbers of each result. If these sequences are used to
> estimate the probabilities, then most sequences will give p = 0.5 for each
> result. This is a well-known result.

Consider a fair dice, and the two outcomes: a six, and the numbers
1-5. According to your argument, the probability of each outcome is
1/2. Clearly something has gone wrong.

The only thing that has gone wrong is that you are using your intuition that there are more integers between 1 and 5 than for the single six. The Hilbert space is still only two dimensional and can be represented by

           |psi> = a|1> + b |0>

If both outcomes are realized on each trial, then after one trial we have branches

         0, and 1.

After 2 trials, we have 4 branches: 00, 01, 10, and 11.
After 3 trials, there are 8 branches: 000, 001, 010, 011, 100, 101, 110, and 111.

After N trials , there are 2^N branches, covering all possible binary sequences of length N. According to the binomial theorem, the distribution of 0s and 1s tends to peak towards equal numbers as N increases. So, for large N, the majority of branches have proportions of 0 and 1 that tend towards 1/2.

This is a simple and indisputable consequence of the law of large numbers. One notes that the outcomes on the 2^N branches are independent of the original weights (coefficients) a and b. Since both outcomes are realized on each trial, the individual weights are of no consequence -- one cannot get a different set of 2^N bit strings by changing the weights. Your intuition that 1-5 and 6 have different weights has led you astray when we are considering only two dimensional outcomes.

Since the majority of sequences for large N have approximately equal numbers of 0s and 1s, the probability for each tends towards 0.5, independent of the presumed weights for each possibility.


What I'm saying is that the probability must depend on both the
anterior and posterior states. In this thought experiment, the
anterior state is one of maximum ignorance, but the posterior states
have uneven weights.


What you are claiming is that the probability should depend on the coefficients a and b in the above general form for a vector in a 2-dimensional Hilbert space. That intuition breaks down when both possibilities are realized on each trial. The development of the state

      |psi> = a|1> + b|0>

is given by

   |psi>|environment> = (a|1> + b|0>)|envirnment> = a|environment_1> + b|environment_0>

and the environment itself is not affected by the coefficients (weights) a and b.

In the 2-dimensional case, the probabilities for each outcome tend to 0.5 for any input vector.

Bruce

John Clark

unread,
Dec 26, 2021, 6:15:07 AM12/26/21
to 'Brent Meeker' via Everything List
On Sat, Dec 25, 2021 at 11:04 PM Brent Meeker <meeke...@gmail.com> wrote:

> To equate probability with self-location there must be a proportionate number of locations.  Otherwise you would have to suppose there is some "weight" of being in a certain branch.  

Not a weight of being, all the observers in all the branches feel equally real regardless of the amplitude of the quantum wave, but you do need to adjust the probability if you want unitarity, and without that "probability" is meaningless. Fortunately there is an obvious way to do that by way of the Born rule.


> Neither of these exist in the bare Schroedinger equation.  I'm not saying the MWI is wrong because it needs these supplementary hypotheses;

The Born Rule is not a hypothesis, it's a proven fact, or at least as close to one as science ever gets. And the quantum wave is not "supplementary", it's what Schrodinger's equation is all about.

 > but I am saying its superiority to simply saying one branch happens is less than obvious.

Every quantum interpretation (except for superdeterminism which believes in ontological certitude not probability) takes the Born Rule for granted, including Many Worlds.

John K Clark   See what's on my new list at  Extropolis
qwf

Lawrence Crowell

unread,
Dec 26, 2021, 7:25:04 AM12/26/21
to Everything List

The Born rule is not a proven result within the postulates or physical axioms of quantum mechanics. It is something that works well in QM, and we take it as almost a type of proven theorem. It is related to the Gleason theorem, so it might in the end be provable.

The attempt by Hossenfelder and Palmer to derive a superdeterminism may lead to interesting results, but maybe very differently from what she thinks. The formalism relies upon p-adic number theory. I think that for Hossenfelder and Palmer to be right it requires that Hilbert’s 10th problem have a solution.  Since superdeterminism is a global law it would require a single consistent algorithm for solving p-adic problems, which are equivalent to Diophantine equations.  Matiyasevich proved a variant of the Gödel theorem which showed global solutions do not exist.

In effect there are limits to systems and observations. In particular, these systems when they observe themselves are akin to Turing machines that encode each other. The universal Turing machine is not able to emulate all machines, such as itself emulating all machines including itself. There is then a type of measurement horizon, which does not permit the encoding of all possible information within the algorithm of the TMs. We may think of this as a sort of horizon that requires a “forcing” of the numerical system. This lack of a universal solution to of p-adic Hossenfelder and Palmer appeal to us a type of forcing that requires imaginary numbers in QM. Something similar occurs with spacetime as built from quantum entanglements. This incompleteness is translated into the occurrence of horizons in spacetime.

There is a lot of course I am glossing over above, but I think nature has these fundamental limits on the capacity for systems of encode themselves. This is I think one element behind QM and GR. With Hossenfelder and Palmer they advance something that appears mathematically false, and because of this they can say there are absolutely determined hidden variables. 

LC

Bruce Kellett

unread,
Dec 27, 2021, 7:53:57 PM12/27/21
to Everything List
On Tue, Dec 21, 2021 at 11:53 AM Jesse Mazer <laser...@gmail.com> wrote:

Personally I still lean towards some version of the MWI being true mainly because you can come up with a toy model with MWI-style splitting that deals with Bell style experiments in a way that preserves locality but doesn't require hidden variables (see https://www.mdpi.com/1099-4300/21/1/87/htm ) but I see it as a sort of work in progress rather than a complete interpretation.

I have had a chance now to look at the paper Jesse refers to here by Brassard et al. As I suspected, it is nothing more than a load of nonsense. They model the correlations in terms of what they call "non-local boxes". This is all very well, but no such boxes are physically realizable, so their argument rather loses its point.

The interesting part of the paper for our discussion, is section 5, in which they give what they consider to be a local explanation of the Bell correlations. Since their non-local boxes are not physical, I will translate their argument into measurements by Alice and Bob on a pair of entangled particles in the singlet state.  One can reproduce the Brassard argument by considering only the case in which Alice and Bob use parallel polarizers. The quantum correlation is then that if Alice measures 'up', Bob necessarily measures 'down'. And if Alice measures 'down', Bob necessarily measures 'up'.

I continue with a quote for page 8 of the paper (translated into spin measurement terms).
"For example, if Alice sees 'up', she splits, and there is a 'parallel' Alice who sees 'down'. Her system can be imagined to carry the following rule: you are allowed to interact with Bob if he saw 'down'. Should this Alice ever come into the presence of a Bob who had seen 'up', she would simply not become aware of his presence and could walk right through him without either one of them noticing anything. Of course, the other Alice, the one who had seen 'down', would be free to shake hands with that Bob."

The paper goes on to elaborate this argument in terms of what happens to Bob after he splits on seeing either 'up' or 'down'.

In their attempt to eliminate non-locality, Brassard et al. have been forced to resort to unvarnished magic. The split Alices and Bobs inhabit different parallel worlds, so what happens when Alice_up meets Bob is that she splits into two copies again: one who sees Bob_up and one who sees Bob_down. Similarly for Alice_down. Brassard et al. are essentially claiming that magically, the pairing of Alice_up with Bob_up can never happen. They give no rationale for this, or any physical explanation as to how this could happen. They simply claim that the pairings up-up and down-down are forbidden and can't happen -- by magic it would see. Of course this magic is local in that it happens only when Alice and Bob meet. But that is no more a physical explanation of the Bell correlations than is the existence of Brassard's "non-local boxes". Of course, the argument they give cannot be generalized to the typical case of non-parallel polarizers either.

The whole paper is a crock of shit!

Bruce
Reply all
Reply to author
Forward
0 new messages